Pdm Pitanja i Odgovori I-i-trp-2

February 9, 2019 | Author: Nikola Jovanovic | Category: N/A
Share Embed Donate


Short Description

Ispitna pitanja...

Description

PITANJA I ODGOVORI

1.Karakteristike oralnog karaktera? Karl Abraham je zapazio uticaj oralnih iskustva na formiranje karaktera. Podvlacio je da oralno zadovoljstvo ne iscezava nikada potpuno I da libidozne gratifikacije vezane za sisanje ostaju u razlicitim oblicima ponasanja odraslog. Srecu se ekstremne varijacije od licnosti obelezenih optimizmom, poverenjem I ljubaznoscu do ispoljavanja zavisnosti, zavisti, neprijateljstva, uznemirenosti prema dozivljaju prvih oralnih iskustva. Obelezja koja se povezuju sa oralnoscu su pasivnost, agresivnost, zavisnost, pohlepnost, samoljubivost, pesimizam, optimizam, slab dodir, apetit, sklonost ka grizenju, pusenju, grickanju noktiju. Ispoljavanje ovih crta licnosti zavisice od stepena ucvrscenja instiktivne kateksije na objekat, u prvom redu libidne I agresivne. Ako ucvrscenje zahvata veliki deo E, ostaje manje E za prelazak u sledece faze razvoja I za nove raspodele.

2.Karakteristike analnog karaktera? Urednost I tacnost I to ne samo u odevanju, vec I u radu, nacinu misljenja, moralu, odnosu prema drugima. Posvojnost, Skrtost, Povisena osujetljivost, Postovanje pravila ponasanja, Dvojnost osecanja I teznji. 3.Dinamika gojaznosti? Uzroci su mnogostruki I cesto udruzeni Konstitucionalni- urodjena prozdrljivost, sklonost gojaznosti, dete je gojazno bez obzira na kolicinu unesene hrane. Hormonski, Psihicki, Drustveni,

Metabolicki. Konstitucionalni cinioci se cesto preplicu sa psihickim, primeceno je das u deca gojaznih roditelja cesce gojazna pa se postavlja pitanje da li je rec o konstituciji ili oponasanju, poistovecivanju. Majke koje na taj nacin pruzaju ljubav detetu. Majke koje na taj nacinsmanjuju svoj osecaj krivice. Pretpostavlja se da upravljanje apetitom, koje je jednom bilo poremeceno, tesko moze da se dovede u red, cak I onda kada je spoljasnja draz ( npr. Uporni zahtevi majke na uzimanju hrane) prestala.

4.Licnost gojaznog deteta, po Brusu? Cesca je kod devojcica I pracena je ubrzanom polnom razlikom, -

manja pokretljivost,

-

manja prilagodljivost,

-

slabija prihvacenost u grupi,

-

manjkavost telesne seme,

-

zbrka u polnom poistovecivanju,

teskoce da opaza I razlikuje drazi iz vlastitog tela, sto je sastavni deo mentalnog ustrojstva, tesko prepoznaje drazi gladi I sitosti, izmenjena predstava o svom telu, njegovim granicama I nedovoljno razlikovanje od tela drugih.

5.Prognoza gojaznosti? 3oblika gojaznosti su Gojaznost bez osecajnih smetnji, Gojaznost kao odgovor na potrese I sukobe, odvajanja, razvod roditelja, operativni zahvati, Trajna gojaznost koja se javlja u okviru dubljih osecajnih poremecaja ( neuroza, psihoza, karaktera) ili gojaznost koja je prvobitnog porekla, a osecajne smetnje su naknadnog porekla. Postepeno se povlaci nakon otklanjanja sukoba.

6.Neutralizacija? Posledica fuzije, mesanja agresivnih I libidioznih teznji, kojom odredjene teznje postaju sposobne da budu prihvacene od svesnog Ega I izvedene preko misica, cula ili sposobne da se pojave u svesti. Ukoliko teznje nisu neutralisane samom fuzijom one bivaju obradjene mehanizmima odbrane koji dovrsavaju neutralizaciju.

7.Regresija u sluzbi Ja? U decijem uzrastu regesija najcesce predstavlja korak nazad radi dva napred, to je nazadovanje u trenutno preteskoj situaciji da bi se sakupile snage za dalju progresiju. Najcesce je koristan cinilac u procesu razvoja. Njeni ucinci su spontano reverzibilni. U decijem uzrastu je cesta, lako prepoznatljiva, ima korisne I socijalno prihvacene modalitete (igra).

8.Ego- sintono I Ego- distono? Ego- sintono- za ponasanje I zelje kazemo da su Ego- sintone kada su saglasne sa subjektovim idealima ili shvatanjima o samome sebi. Ego- disono- za ponasanje I zelje kazemo das u Ego- distone kada nisu saglasne sa subjektovim idealima ili shvatanjima o samome sebi, odnose se na vrednosni sud koji donosi sam subject.

9.Autoplasticna I aloplasticna adaptacija? Termine je uveo F. Aleksander 1930 godine, da bi podvukao razliku izmedju adaptivnih odgovora koji menjaju sredinu-aloplasticnih I onih koji menjaju self- autoplasticnih. Mehanizmi odbrane psihoneuroze su klasicni primeri autoplasticne adaptacije.

Gde se javlja strah od odvajanja? Kod simbiotske psihoze, kod skolske fobije, ozivljava se I ponavlja strah od odvajanja, Moze kod slepih I gluvih zato sto su previse zavisni od drugih.

Deca prevazilaze strah kroz? Umereno zastrasujuce prilike, osobe, igre, price koje ih uzbudjuju, umereno plase I podsticu na mastu, predvidjanje I delatnosti.

10.Restitucija? Odbrambeni process smanjivanja krivice iskupljivanjem prema ambivalentno investiranom objektu. Proces kojim shizofreni ili paranoidni pacijenti stvaraju sumanutosti koje im omogucavaju vracanje osecanja znacaja.

Deca ponavljanjem prevazilaze? Sveza povredjujuca I osujecujuca iskuistva, pokazuju teznju da se ponavljaju kroz snove, igru, pricu. Kroz ponavljanje povrede u snu psiholoski aparat uci kako da ovlada strpnjom koju povreda izaziva. Neprijatna snovna iskustva mogu oziveti stare potisnute sukobe. Ponavljanja u snovima, igri, prici, imaju funkciju da se kako tako ovlada sadasnjom povredom. Za vreme povredjivanja, detetovo ja bilo je pasivno I bespomocno, a sada, prilikom ponavljanja povrede u snu ono je delatno, mislece I sposobno da predvidi sta ce se dogoditi, ako se to isto ponovi. Tako psihicki aparat uci kako da ovlada strepnjom koju povreda izaziva.

11.Reparacija? Proces ( mehanizam odbrane ) putem koga se smanjuje osecanje krivice tako sto se u dobro pretvara ono sto se zamislja da je lose ucinjeno ambivalentno investiranom objektu. Proces obnavljanja unutrasnjeg objekta koji je bio razoren u fantaziji. U Klajnijanskoj teoriji sve kreativne aktivnosti se smatraju reparativnim, u skladu sa stanovistem da reparacija predstavlja normalan process putem koga jedinka razresava inherentnu ambivalenciju prema objektima. 12.Sta je aleksitimija? Nesposobnost da se protumace I na simbolican nacin izraze osecanja. 13.Najefikasniji lek kod hiperkinetickog sindroma?

Ritalin Psihostimulansi ( ritamin, deksedin, cylert ), Triciklicni antidepresivi ( norpamin, tofranil ), Ostali lekovi ( klonidin, tirodazin ). 14.Ko uvodi u psihoanalizu aktivno- pasivno? Frojd je sirokko koristio ideju postojanja polariteta izmedju aktivnosti I pasivnosti. Po klasicnom psihoanalitickom tumacenju Aktivnost se odnosila na Maskulinost, Agresiju, Sadizam, Vojerizam, A pasivnost na Femininost, Submisivnost, Mazohizam, Egzibicionizam. Ova podela se komplikovala daljom pretpostavkom da instikti mogu da pretrpe obrtanje u suprotno, narocito aktivni instikti mogu da postanu pasivni, pri cemu su se kao primer najcesce izdvajali sadizam I voajerizam. U kasnijem razvoju psihoanalize jednacina aktivno- maskulini, a narocito pasivno- feminine, pretrpela je niz izmena.

15.Razvojni strahovi, kada se javljaju? Ne nastaju usled spoljasnje ugrozenosti, vec usled nezadovoljenih nagonskih teznji odnosno saline instiktivnih pokretaca I sastavni su deo psihickog razvoja. Javljaju se kao posledica bioloske I psiholoske nezrelosti odnosno zavisnosti od majke, odraslih. Razvojni strahovi su Strah od komadanja, Strah od odvajanja, Strah od prljanja,

Kastracioni strah, Strah od kazne Nad- ja. Strah od komadanja- u prvoj polovini prve godine usled pretnje moguceg gubitka kontrole nagonskih teznji, odnosno gubitka tek labavo uspostavljenog integriteta Ja u situaciji kada jos nisu uspostavljene telesne I mentalne granice Ja, a investiranost psihickom energijom je jos na nivou parciijalnih objekata. Strah od odvajanja- separacioni, anakliticki oko 8 meseca detetovog zivota, u vreme kada majku pocinje da dozivljava, prepoznaje kao zasebnu licnost. Traje do oko 16 meseca kada dete izrasta. Iz ovog straha, koji ostaje manje, vise prepoznatljiv I ko sastavni deo covekovog emocionalnog zivota. U decijem uzrastu on se sasvim lako vezuje za kaznu zbog realno ucinjene ili samo zamisljene neposlusnosti, prestupa, nacinjene stete. Strah od prljanja- Razvojno je vezan za drugu godinu zivota, kada preko poistovecenja sa roditeljskim zabranama, dete razvija mogucnosti pretvaranja prvobitne nagonske potrebe za prljanjem u njenu suprotnost. Kastracioni strah- strah Edipalnog perioda razvoja odnosa prema objektu I u raznim varijantama poprima vid strasljivog iscekivanja kazne zbog nekog nedozvoljenog ponasanja sopstvene agresije, nedozvoljenih neznih osecanja, ljubomore… Strah od kazne Nad- ja- oko 5-6 god, u vreme kada je vec formirana ova istanca licnosti. Moze da bude veoma jak kada je dete prekrsilo eticke odnosno moralne norme koje je usvojilo putem poistovecivanja sa roditeljima. Nastaju iz nezadovoljenih teznji. Deca se tokom vremena sukobljavaju sa izvesnim situacijama u kojima prepoznaju svoje razvojne strahove u raznim varijantama. Po svom karakteru I strukturi srodni su fobicnom stanju odraslog, jer nastaju u vreme pre nastanka genitalnog primate, pa zato nisu u stanju da se organiizuju I utope u sve one nacine odbrane koji stoje na raspolozenju zreloj licnosti.

16.Interminentnost- alternativnost? Interminentnost- neki oblici ponasanja mogu da se pojave u odredjenom uzrastu, a zatim se izgube I posle izvesnog vremena opet pojave. Intervali gubljenja se skracuju sve dok se ponasanje potpuno na ucvrsti.

Alternativnost- pojava da se u nekim periodima neke funkcije ili grupe funkcija razvijaju brze I vise, na racun drugih, koje privremeno stagniraju.

17.Zbog cega se javlja fiksacija? Proces putem koga osoba postaje, ostaje ambivalentno vezana za neki objekat, pri cemu je taj objekat bio odgovarajuci ranijem stupnju razvoja. Fiksacija- znak nedovoljno uspesnog razvoja kroz faze razvoja odnosa prema objektu. Osoba sa fiksacijom Ima tendenciju da pribegava infaltivnim semama ponasanja ili da nazaduje na ove vidove ponasanja kada je pod stresom, Da prisilno bira objkte na osnovu njihove slicnosti sa objektom za koji je fiksirana, Trpi zbog nedostatka raspolozive psihicke energije usled njene ulozenosti u taj objekat. Javlja se zbog toga sto odredjeni dogadjaj suvise jako obelezi razvojni stadij, pa je prelaz u sledeci stadij otezan. 18.Karateristike osoba koje su pokusale samoubistvo? Deca I omladina koji pokusavaju samoubistvo ne ispoljavaju posebna psihopatoloska obelezja na osnovu kojih bi se mogao dijagnostikovati samoubilacki poremecaj kao zasebna klinicka Celina. Sledeca obelezja licnosti ove podgrupe mladih mogla bi biti povezana sa samoubilackim ponasanjem Smanjena sposobnost za govorno saopstavanje unutrasnjih sadrzaja I misli, Sklonost ka prelasku na radnju, Nemogucnost mentalne obrade telesne napetosti, Preteranokoriscenje mehanizma intelektualizacije I racionalizacije koji ne uspevaju da sprece agresivne teznje I ublaze mrzovoljnost, samoljubivost I strepnju, zalost I neprijateljstvo prema sebi I prema svetu. 19.Sta je mitomanija? Laganje, neistinito prikazivanje stvarnosti. Podela Neduzno laganje, Mitomansko, Prestupnicko laganje.

Na starijem uzrastu dete pocinje da razlikuje stvarno od nestvarnog I uskoro njegova istina postaje realnija I opstija. Zbog snaznih osujecenja u stvarnom svetu I sa zeljom da se zadovoljenja nadoknade, jedna grupa dece, mladih pa I odraslih nadopunjuje nepozeljna iskustva I dozivljaje izmisljenim I za njega uvek pozeljnim sadrzajima. Ovi mitomanski sadrzaji treba da ispune neispunjene zelje I ocekivanja I nadoknade razocaranja I osujecenja.

20. Latencija? Faza mirovanja Stadijum mirovanja, Nakon Edipalne, falusne faze, Pocinje nakon 5-6 god, Traje do ulaska u pubertet, Unistenje Edipalnog slozajai oslobadjanje energije potrebne za druge delatnosti Ja, na primer intelektualne. Faza u kojoj je libidna energija oslobodjena za intelektualne delatnosti, za stvaranje prijateljstva, bavljenje umetnoscu I drugim visim delatnostima. U ovoj fazi su mehanizmi potiskivanja I poistovecivanja posebno zivi. Posebno mesto zauzimaju reaktivne formacije koje stvaraju snazna protivulaganja ( kontrainvestiranja ). 21 .5 nekonfliktnih funkcija po Frojdu? Opazanje- na sta ukazuje mogucnost razlikovanja voljenog objekta od svih ostalih, Voljne aktivnosti- omogucene dovoljno razvijenim motornim sposobnostima, Rasudjivanje- koje je omoguceno postojecim tragovima pamcenja, Ranog razvoja nacela realnosti- na sta ukazuje pocetna poslusnost zabranama I pravilima ponasanja, Sinteticka funkcija Ja- obezbedjuje osnovu za integraciju dobrog I zlog objekta, odnosno fuziju ( I neutralizaciju ) libido I agresivnosti. 22. Mladalacka kriza?

Smetnje prilagodjavanja u mladalackom dobu. Cesto moze da otvori I put dubljim razgradnjama I patoloskom razvoju. Nema jedinstvenog nacina ispoljavanja. Moze da poprimi obelezja svih drugih smetnji I poremecaja I biti uvod u druge poremecaje kao sto su neuroze, psihoze, psihopatije I nedrustvena ponasanja. Nagle telesne promene. Bitno je izmenjen osecajni zivot. Smanjenje intelektualnih I drugih zanimanja, intelektualna kocenja, slab uspeh u skoli. Povecanje misicne mase. Promena oblika I izgleda delova tela. Fizioloske, hormonske promene. Pojava polnih oznaka. Razvoj apstraktnog misljenja dostize visi stepen u ovom uzrastu. Promena stavova prema mladoj osobi, roditelji isto dozivljavaju krizu. Menjaju se odnosi sa roditeljima, postaju labavi, lomljivi, cesce ispunjeni dvojbom I neprijateljstvom. Protivurecna osecanja ljubavi I mrznje, tuge, depresije, strepnje od odvajanja.

23. Fobija od defeciranja? Ispoljava se kao jak strah, pracen fizioloskim promenama. Javlja se onda kada dete ima potrebu za praznjenjem I kada oseti prva grcenja cmarnih I drugih misica ili pokreta creva ili kada roditelji pokusavaju ili nagovestavaju stavljanje na sud. Dete je zaplaseno, placljivo, na sve nacine se bori da izbegne ovu radnju. Obelezja licnosti su Povisena plasljivost, povisena zavisnost od roditelja, nesigurnost, pasivnost, zakocena I pritajena agresivnost, osecajna nezrelost, smanjena podnosljivost na zahteve okoline, dvojnost osecanja prema ocu. Uzroci su mnogostruki Uloga roditelja Rano stavljanje na nocni sud Prisila I stega, Preterano zasticavanje deteta I zadrzavanje u detinjastom polozaju u porodicnoj grupi, Neprijateljstvo u odnosima, Stroge kazne na propuste, Iskrivljena opazanja I fantazmi o samom izmetu I cinu.

Organski cinioci Ostecena cns,

Organski megakouon, Mala telesna tezina I visina, Polipi creva, Bolesti crevnog trakta. Prognoza I tok Povoljan odnos izmedju roditelja I deteta, nakon otkrivanja I otklanjanja stave koji izaziva poremecaj, ima pozitivan ucinak na tok I prognozu. 24. Koji su mehanizmi odbrane kod fobija? Premestanje, Sazimanje, Simbolizacija, Projekcija, Opasnost biva premestena u spoljni svet objekata daleko od unutrasnje stvarnosti. Premestanjem se prenosienergija sa jednog objekta na drugi, u slucaju fobije na spoljasnji. Projekcijom se opasnost, koja je sada spoljasnja, moze izbeci, sto fobicna osoba I cini. Fobije se lakse javljaju kod onih kod kojih potiskivanje nije dovoljno jako I kod kojih su se mehanizmi projekcije I premestanja zadrzali duze. Mehanizmi odbrane kod opsesivno- kompulsivnih neuroza su Potiskivanje, Izdvajanje, Nazadovanje, Intelektualizacija, Reaktivna formacija, Racionalizacija, Psihodinamika, kada potiskivanje otkaze, postoji opasnost da instiktivneteznje dopru do presvesne ravni. Ja da bi ocuvalo celovitost, pribegava nazadovanju, kako bi osnazeno bolje upravljalo prvobitnim teznjama. Savremeni sukobi pomeraju se na preedipnu ravan. Ja u nazadovanju bolje upravlja preedipnim sukobima.

25. Na kom uzrastu se javljaju fobije, po Bakeru? 11- 14 godine.

26. Dinamicko, ekonomsko I topicko glediste? Dinamicko- sve motivacije ljudskog ponasanja su konfliktne I premestanje od principa zadovoljstva ka principu realnosti predstavlja jedan od vidova prilagodjavanja cija dinamika oscilira izmedju prevage direktnog zadovoljenja pulzija I osecanja vitalne sigurnosti, ili bolje receno, prevage ono I ja. Ekonomsko- pristupa psihickim fenomenima sa kvantitativnog stanovista prisutnih snaga. Problemi su postavljeni u izrazima pulzione energije ili energije osecajnog investiranja. Topicko- posmatra probleme strukture psihickog aparata. Razmatra suprostavljanje svesnonesvesno, odnos izmedju unstanca licnosti zbog uloge koju ove igraju u konfliktu I nacina na koji posreduju u formiranju licnosti Jednom pojmu, kao potiskivanju, se moze pristupiti sa gledista topike, ali on otkriva I problem dinamike I ekonomike.

27. Razlika nocnog straha I fobije? Nocni strah se razlikuje od fobija po tome sto odbrane Ja u prvom redu potiskivanje, uspevaju da sprece prodiranje nesvesnih sadrzaja I sukoba u budnom stanju.

28. Nad- Ja muskaraca I zena? Kod muskaraca Edipov kompleks nije bio jednostavno potisnut, nego se rasprsnuo pod udarom pretnje kastracije. Njegova libidozna investiranja su napustena, lisena seksualnosti, sublimirana s jedne strane njegovi objekti su inkorporirani u Jag de obrazuju jezgro Nad- Ja. U idealnom slucaju Edipov kompleks ne ostaje da zivi cak ni u nesvesnom, buduci da je Nad- Ja preuzeo njegovo nasledje. Kod zena zbog toga sto ona moze malo- pomalo da napusti Edipov kompleks I dag a resi potiskivanjem, posledice ce biti sasvim razlicite. Nad- Ja zene nece nikad biti tako neumoljiv, bezlican, odvojen od svih osecajnih potreba kao kkod muskaraca. U posebnosti izgradjivanja zenskog Nad- Ja Frojd vidi poreklo izvesnih crta svojstvenih zeni, manju potcinjenost velikim nevoljama zivota. Njene odluke ostaju podredjene neznim I neprijateljskim osecanjima, ona je manje osetljiva na pojmove pravde I pravo.

29. Razlika konverzija I fobija? Slicnosti- u obema se koriste mehanizmi Premestanja, Sazimanja, Simbolizacije, Projekcije. Pacijenti I fobicni I histericni projektuju izdanke svojih nesvesnih fantazija u nameri da se oslobode neizdrzive napetosti I strepnje. Razlike- fobicno dete, omladinac svoje fantazije projektuje u spoljni svet, objekte I okolnosti, pa opasnost ‘’kao da “ dolazi spolja I kao da je jasna razlika izmedju unutrasnje I spoljasnje delatnosti. U konverziji dete, omladinac svoje fantazije projektuje uglavnom u svoje telo, kao da postoji razlika izmedju unutrasnje I telesne stvarnosti.

30. Obelezja licnostisa skolskom fobijom? povisena nesigurnost, plasljivost, povisena povredljivost, sklonost ka nazadovanjima I projekcijama, sklonost ka depresivnim I fobicnim odgovorima, zavisnost, samoljubivost. 31. Odgovori dece na razvod? Pod najpovoljnijim okolnostima agresijom, poricanjem gubitka, udaljavanjem od roditelja I strastveno predavanje sportu, muzici, ucenju. Koristi situaciju da dobije novac, izlaske, opravdanja, neuspeh u skoli. Mladja deca – smetnje apetita, spavanja, deficiranja, bes, nemir, Starija deca- osecajna hladnoca, strepnja I fobije, osecaj krivice, depresija, pokusaji samoubistva. Pod nepovoljnim okolnostima, najnepovoljnijim, deca I omladina koji su ispoljavali smetnje prilagodjavanja mogu ispoljiti neuroticne I psihoticne odgovore I stanja, nedrustvena ponasanja, psihosomatska oboljenja. Ponekada je bolji dobar razvod nego los brak.

32. Sisanje prsta? Normalno do nekoliko meseci. Do puberteta I kasnije znak je usporenog razvoja telesne seme, nazadovanja I fiksacije, usporenog razvoja Ja, produzene autoerotske faze.

33. Pojam inkorporacije? Ponekad se koristi kao sinonim za internalizaciju ili introjekciju. Odnosi se samo na fantazam unosenja spoljasnjeg objekta. Fantazmi inkorporacije su tipicno oralni, gutanje objekata, ali mogu da se odnose I na druge otvore tela, oko, uvo.

34. Mehanizmi odbrane kod nocnog straha? Pomeranje, Projekcija u san, Poistovecivanje sa agresorom, Mehanizmi koji se vidjaju I kod fobija, pa neki autori nocni strah izjednacavaju sa fobijama.

35. Konstitucionalna datost? Je izvesno stanje mentalnog funkcionisanja koje osposobljava osobu, vise, manje, da u slucaju potrebe izdrzi frustracije, stress I da integrise tokove sazrevanja.

36. 2 funcije u napadu nocnog straha? Da se zadovolji unutrasnja I delimicno zabranjena zelja, na primer privuce paznja neke osobe, Da se izdrzi kazna zabranjene zelje, trpnja od straha, 37. U kojoj fazi spavanja se javljaju nocni strahovi?

Oko 70- 80 % nocnih strahova nailazi u prvoj fazi nocnog spavanja, u prvoj fazi NREM spavanja. II I IV faza sporog spavanja, nocni strah. REM faza, nocna mora.

38. Strah od smrti? Dete se prvi put sa smrcu susrece u susretu sa mrtvim zivotinjama. Brani se povlacenjem, udaljavanjem I privijanjem uz odraslu osobu. Ovi odgovori se javljaju pre nego sto dete ima svest o smrti. Dete mladje od 3- 4 godine nema izgradjen pojam o smrti, shvata ga kao odlazak umrle osobe iz kuce. 7- 8 godina postoji I kao cinjenica, ali se jos uvek ne uvidja mogucnost vlastite smrti. 8- 9 godina- shvataju da su oni smrtni. Javlja se nocna mora, strah, strah od odvajanja od stvarnosti.

39. Uzroci I psihodinamika smetnji spavanja? Psihicki uzroci Sukobi spoljnjeg I unutrasnjeg porekla koji izazivaju strah. Spoljni sukobi I neprijatna iskustva narocito kod mladje, ali I starije dece, iz prethodnih dana, grube kazne, napustanja, osujecenja, nneuspeha u skoli. Mogu biti neposredni uzrok smetnjama spavanja. Neprijatna iskustva I sveze povrede preplavljuju Ja uzbudjenjima koje ne moze da sadrzi I poveze, te ove ometaju njegove funkcije I odrazavanje spavanja. Uzroci I psihodinamika smetnji spavanja mogu biti I mnogo slozeniji I dublji I voditi poreklo iz neresenih sukoba, svojstvenih svakoj fazi razvoja. Nezrelost CNS-a. 40. Pijaze, vrste snova kod dece? Snovi koji ispunjavaju nezadovoljene zelje I potrebe, deca cesto sanjaju kako primaju poklone, putuju, Snovi kroz koje se ostvaruju neostvarene zelje bez vidljive simbolike,

Snovi koji svesno pretstavljaju nesto drugo, Snovi koji predstavljaju neki neprijatan dogadjaj, ali sa prijatnim ishodom, kao u igri, Kosmarni snovi ispunjeni strahovima iz kojih dete moze da izvlaci zadovoljstvo ili su bolni, Snovi kaznjavanja ili samokaznjavanja, Snovi koji su izraz organskog podrazaja, 41. Bruksomanija? Obredi uspavljivanja 2- 3 godine. Skrgutanje zubima- bruksomanija, Govorenje u spavanju- somnilogija, Smejanje tokom spavanja, Pokreti kao sto su ritmije glavom I telom koji se cesto javljaju zajedno I pre 3. Godine, Predstavljaju nacine rasterecenja od povisene napetosti I strepnje. 42. Malinovski, Edipov kompleks? Malinovski insistira na kulturnoim relativizmu osnovnih kompleksa oslanjajuci se na proucavanje Melanezijskog stanovnistva Trobrijanskih ostrva. Malinovski smatra da Edipov kompleks moze postojati samo u drustvu patrijahalnog tipa, dok je na ovim ostrvima, gde se porodica struktuira preko matrilinearnog tipa, ocev autoritet prenet na ujaka. U takvoj drustvenoj strukturi Edipov kompleks je premesten u kompleksnu matrilinearnu situaciju.

43.Koji poremecaji su cesci kod slepe dece nego kod zdrave dece I zasto? Kod slabovidih ispitaniika prisutne su depresivne teznje, strepnja I nevoljno umokravanje, 6%, a medju slepim ispitanicima fobicne teznje, konverzivno- histericne teznje i nevoljno umokravanje, 10%,

Opsta I posebna neuroticnost slepih I slabovidih ima ista obeklezja kao I neuroticnost dece I omladine koja vide. Organizovane I ustaljene neuroze, kao I psihoze slepih I slabovidih isto toliko su ceste, imaju ista obelezja, uzroke I dinamiku kao I medju videcom decom I omladinom. Sa polaskom u skolu slepa I slabovida deca mogu da ispolje teskoce prilagodjavanja zbog pojacanih zahteva skole, ulaska u grupu drugova I povecane svesti o svom nedostatku. Slepa deca se snalaze u stvarnosti obicno preko cula sluha I dodira. Odsustvo zvukova I telesnih dodira za njih je istovetno sa odsustvom objekta. U tom praznom prostoru lako se iskusi strepnja I gubi se orjentacija u prostoru. Ova deca su vise izdvojena od drugih osoba I nemaju dovoljna energetska rasterecenja.

44.Telesno Ja u snovima, po Federnu? Osecanje telesnog Ja slozeno I ukljucuje sva motorna I senzorna secanja koja se ticu same osobe. Moze da se smatra delom osecanja mentalnog Ja. Kod naglog padanja u san osecanje telesnog Ja nestaje pre osecanja mentalnog Ja. Telesno Ja moze sasvim nestati tokom padanja u san I moze biti nanovo investirano I probudjeno od strane mentalnog Ja, koje ostaje budno. Na taj nacin mozemo uspeti u dobrovoljnom odlaganju spavanja. Kod normalnog procesa budjenja telesno I I mentalno Ja se bude gotovo istovremeno. U snovima sa karakteristicnim senzacijama, bolnim, kao I neprijatnim, telesno osecanje Ja je uvek pojacano, ali cesto nepotpuno. Ovo je osecanje I naglaseno I potpuno u snovima letenja I plivanja, koji su praceni istaknutim osecajem dobrobiti. Telesno osecanje Ja nije identicno sa telesnom semom, vec moze da bude uze I sire od nje, zavisno od kvaliteta I kvantiteta katekse ulozenosti Ja. Telesno osecanje Ja ukljucuje motorna I culna osecanja u vezi sa sopstvenom licnoscu. Ja prelazi iz jednog stanja u drugo, ali uvek uz dozivljaj jedinstva I trajanja koji se tokom privremenih prekida tokom sna ponovo uspostavlja.

45.Prilagodjeni I neprilagodjeni debili?

Umni razvoj ne prelazi uzrast deteta od 10 godina, IQ se krece od 50- 70, Prilagodjeni skladni Ne ispoljavaju veca odstupanja u odnosu na osobe normalne inteligencije, Uspevaju da koriste svoje sposobnosti za sticanje znanja, Razmerno svojim intelektualnim sposobnostima dobro se prilagodjavaju porodici I siroj sredini i Odgovarajuce se profesionalno osposobe, Neprilagodjeni neskladni Ispoljavaju znacajne osecajne smetnje I teskoce drustvenog prilagodjavanja, Vidjaju se apaticni, uznemireni, nepostojani,detinjasti, osecajni, manicni I drugi tipovi, Ego je slabiji, lomljiv, Mehanizmi odbrane su primitivniji, Poistovecivanja su cesce zbrkana, zbrka izmedju zelja I dozivljenog je ocigledna, procena stvarnosti je nepotpuna, Povisena agresivnost, Neprihvacenost od okoline. 46.Federn, derealizacija, depersonalizacija? Derealizacija- Granica Ja predstavlja periferni culni organ koji stoji izmedju Ja I Ne- Ja, a cija karakternost odredjuje zdravlje Ja, licnosti, omogucavajuci razlikovanje izmedkju realnog I nerealnog. Kada spoljasnja granica Ja izgubi svoju kateksu, spoljasnji objekti se dozivljavaju kao strain, nepoznati, bezivotni, nerealni. Depersonalizacija- povecanje katekse, ulozenosti, daje objektima realnost, zivost. Slabljenje unutrasnjih granica dovodi do halucinacija koje se dozivljavaju kao realne, zato sto poticu od mentalnih stimulus koji su usli u svest, a da pri tome nisu ulozene, investirane od Ja. Slabost Ja je posledica nedostatka izvora kateksi od kojih libido igra najznacajniju ulogu. Ostecenje Ja I njegovih funkcija nisu uslovljeni pojacavanjem, obogacenjem ulozenosti Jan a racun objekta libido, vec njihovim slabljenjem, odnosno lezijom granica Ja.

47.Uzroci nastajanja mutizma? Nove I zastrasujuce okolnosti, Prisustvo odraslih, Stanja iznenadjenja,

Za vreme ispita, Cesce je kod dece sa smetnjama artikulacije. Dete izvlaci sekundarnu dobit, ne odgovara u skoli, izbegava odnos sa roditeljima koji kaznjavaju,

48.Slicnosti I razlike mehanizma odbrane, po Walderu? Zajednicke crte mehanizma odbrane Oni teze da iz svesti isceznu svi mucni afekti ili afekti koji se cesto zadovoljavaju, Oni su cesto specificni za odredjenu osobu, pronalaze se uvek kod iste osobe pre nego neki drugi, Mehanizmi odbrane dovode do pojave substitutivnih gratifikacija, koje su neprilagodjene svom prvobitnom objektu, ili koje zauzimaju mesto u obliku neuroticnih simptoma, nemoguce gratifikacije. 49.Kognitivno- bihejvioralno terapija kod depresije? Prema teorijiucenja depresija je izazvananedostatkom potkrepljenja I prisustvom averzivnih stimulus. Ako je izgubljen vazan objekat koji je nadzirao veliki broj pokusaja deteta, gubi se potkrpljivac I ova ponasanja se prekidaju. S druge strane bespomocnost je naucen stav o zivotu koji je tesno povezan sa depresijom. Agresija je smanjena kod depresivnih po teoriji ucenja. Kognitivno objasnjenje depresije Etiologija depresije- izopacene kognicije, percepcije. Depresivna deca sebe vide kao Negativne, Neadekvatne, Beznadezne. Tuga I ocaj dolaze iz negativnih pogleda na sebe I svet.

50.Uzroci mastrubacije? Dete dosta rano otkrije da neki delovi tela obezbedjuju zadovoljstvo, bilo da ioh dodiruju drugi ili ono samo.

Tokom prve godine dolazi do stezanja butina, ljuljanja, trljanja polnih delova rukama, koji obezbedjuju podrazaj genitalnog aparata kod dece. Dodirivanje polnih organa, u prvo vreme slucajnoi, a kasnije sa ciljem, ima funkciju ispitivanja tela, da bi nakon otkrivanja zadovoljstva postal namerna radnja koja obezbedjuje polno zadovoljstvo. Sek. U falusnoj fazi, 3- 5 godine.

51.Kako se odredjuje snaga Ega? Obracajuci se snazi Ja, kvantitativnoj datosti, Ana Frojd razmatra uspesnost Ja u njegovom zadatku da se odupre instinktivnim pulzijama. Pred instinktivnim pulzijama Ja treba da se ponasa kao instance koja regulise zelju. Nakon neposredne vladavine instikta dete stice mogucnost da radi ostvarivanja svojih zelja iskoristi spoljasnji svet. Identifikacijom sa roditeljima Ja osobe pokusava da upravlja svojim zeljama I dap red pulzijama prihvati kriticni stav koji odgovara internalizovanim roditeljskim zahtevima. Princip zadovoljstva, malo po malo zamenjuje princip realnosti koji tezi da izmiri zelje sa zahtevima iz spoljasnjosti. Ova funkcija posrednika pada na Ja koje se tako nalazi u konfuznoj poziciji sa Ono. Ana Frojd u uspostavljanju ravnoteze izmedju pulzija Ono I snaga Ja vidi bitan uslov za harmonican razvoj licnosti.

52.Uloga bajki kod dece? Bajke su kod dece posebno prihvacene na prelazu iz konkretnog na formalno- operacionalni period, oko 10 godine, jer odgovara razvoju saznajnih funkcija. One pomazu razresavanje mnogih emocionalnih problema kod dece, posebno strahova. Nesvesni sadrzaji, koji se pojavljuju u snovima, u bajkama bivaju jasnije operacionalisani I svesno dozivljeni, obicno sa srecnim I povoljnim ishodom. Psihoanaliticko tumacenje bajki je blisko tumacenju mitova I snova. One se takodje shvataju kao rad nesvesnog kroz odredjene simbole I njihove relacije.

53.Psihodinamika I uzroci preteranih mastrubacija? Patoloska mastrubacija ima sledece odlike

Ucestala, Prisilna obelezja, Iskljucivi vid polnog zadovoljenja, Zavisnost koja odvlaci veliku ili svu kolicinu libido.

Patoloska je jer otezava I onemogucava dalji razvoj polnosti, pa I dalji razvoj licnosti, pa I prilagodjavanja. U mladosti ima dublje uzroke Preterano podsticanje, zavodjenje I prerano upucivanje na polnost od strane roditelja I drugih osoba, Slabi odnosi sa roditeljima I drugovima. Izazivaju Jaka strahovanja I strepnju, Osecanje krivice I stida, Hipohondricne tegobe, Dalje slabljenje drustvenih veza. 54.Ispoljavanje agresivnosti na oralnom, analnom, falusnom stadijumu I kasnije? Oralni stadijum Prazni se preko usta, grizenje, gutanje, prozdiranje, ujedanje, pljuvanje, a na visoj ravni razvoja kroz govor,

Analni stadijum Prljanje, defekacija, ispustanje vetrova, tvrdoglavost, svojevoljnost, mucenje, zlocestost,

Falusni stadijum Na manje neposredan I telesni nacin, a vise kroz osecajna neprijateljstva, Kasnije Simbolicno kroz igru, svakodnevne delatnosti I odnose, sport, stvaralastvo, rad, nauku,

55.Karakteristike I psihodinamika dece sa astmom? Najcesce psihosomatsko oboljenjedece I omladine. Psihodinamika Napadi sa promenama u ritmu udisaja I izdisaja zbog grca bronhiola I poremecaja u lucenju sluzokoze disajnih organa, Jako dozivljavanje straha od ugusenja Karakteristike Povisena strepnja I zavisnost, Nepoverljivost I napetost, Slabije ustrojstvo Ja 56.Uzroci postraumatskog stres sindroma? Karakteristika ptss je razvoj simptoma koji smetaju osobi, ali se mogu izbeci, a javljaju se nakon izlaganja nekom traumaticnom dogadjaju. Traumatican dogadjaj se definise pomocu Izlaganja smrtnoj opasnosti, ugrozavanju fizickog integriteta licnosti. Osoba ne mora biti ucesnik u dogadjaju, mada je potrebno odredjeno licno iskustvo, posmatranje traumaticnog dogadjaja, Dozivljeno iskustvo izaziiva intenzivan strah, uzas, bespomocnost, dezorganizovana ponasanja ili suzdrzana, Ova poslednja 2 se javljaju samo kod dece. Simptomi su uglavnom isti kod odraslih I dece, s tim da se kod odraslih vise osecaju na kognitivnom planu, a kod dece na bihejvioralnom. Da bi se postavila dijagnoza potrebno je da simptomi postoje bar 1 mesec. Ako se simptomi jave odmah nakon nesrece 2 nedelje, 4 nedelje, radi se o akutnom postraumatskom sindromu. Lecenje Bihejvioralne tehnike, desenzibilizacija, Psihoterapija, terapija fokusiranja na traumu, 57.Alergijski tip licnosti?

Potiskivanje agresivnosti, Poricanje sukoba, Preterano koriscenje pomeranja, Ne ispoljavaju strah od nepoznatih osoba, Stvaranje alergijske licnosti dovodi se u vezu, kako sa urodjenim psihickim, tako I urodjenim alergijskim ciniocima, 58.Prisila ponavljanja? Poznati nacin decijeg prevazilazenja straha je prisila ponavljanja. Sastoji se u tome sto kroz vise puta ponovljenu situaciju koja zastrasuje, a deca to cine kroz igru, kroz pricu, crtez, salu, dete se upoznaje sa zastrasujucom situacijom, predmetom ili osobom, snaga strasnog se smanjuje, dete “manipulise” strasnim time sto ono ponavlja situaciju I tako postepeno ovladava strahom I prevazilazi ga. 59.Kad se javljaju funkcije Ega, Hartmanu? Ono I Ja nastaju kao nezavisni sklopovi iz prvenstveno nediferenciraneosnove, biofizioloske, matrice. Ja se ne razvija iskljucivoucenjem I sticanjem iskustava, vec je delimicno autonomno I poseduje svoje boloske korene u uodjenim procesima koji su analogni fizioloskim procesima. Samim tim, nisu sve funkcije Ega posledica sukoba instinktivnih pokretaca sa stvarnoscu, vec funkcije , pamcenje, opazanje, postoje pre sukoba I nezavisne su od njega, odnosno uodjene su I nekonfliktne. Iz prvobitno nediferenciranog stanja ne razvija se samo Ono, vec I autonomno Jas a svojim funkcijama, primarne autonomije. Vremenom ove prijatne aktivnosti mogu da budu ukljucene u system Ono, da se koriste u sluzbi Ono, a isto I u sluzbi Nad- Ja, govor kao ironija, Pored primarne postoji I sekundarna autonomija, posto oblici ponasanja mogu od sredstva da postanu cilj za sebe. Svaka navika koja se stice kao odbrana Ja moze da razvije svoju sekundarnu autonomiju koja obezbedjuje sebi opstanak I onda kada potreba za odbranom vise ne postoji, preterana urednost koja ostaje I nakon nestanka strepnje koja ju je uslovila. Rec je o odvajanju jednog procesa koji se razvija u sluzbi instiktivnog pokretaca od tog istog pokretaca I njegovog funkcionisanja kao da je sam sebi cilj. Dolazi do promene u funkciji. Ovaj pojam obezbedjuje razumevanje odbrana, prilagodjavanja I efikasnosti Ja.

60.Sklopovi licnosti podlozni psihosomatskim oboljenjima, po Krajsleru? Neuroze ponasanja, Nedovoljno organizovane licnosti, Depresivna deca, omladina, Alergijski sklop licnosti. 61.Sta se pre javlja psihosomatski poremecaj ili neuroza? Psihosomatski poremecaji su stariji od neuroticnih, jer se javljaju u ranom detinjstvu pre stvaranja neuroticnog sukoba izmedju delova licnosti. Psihosomatski poremecaj je neposredni izraz afektivnog trpljenja koji u pocetku menja funkciju, a zatim I sklop organa.

62.Nacini reagovanja dece na odbijanje od dojke po M.Ribl? Dva tipa Reakcija obelezena “negativizmom”sa odbijanjem grudi, gubitkom apetita, misicnom krutoscu, povrsnim disanjem, Nazvan “ depresivni tip” oznacen je ravnodusnoscu, misicnom mlitavoscu, bledilom I stomacno crevnim poremecajima. 63.Uzroci anoreksije, razlika primarna I sekundarna? Uzroci organski, psihicki, psihosomatski, kulturni, Jedan od kljucnih uzroka je u izmenjenim odnosima prema sopstvenom telu, Primarna, rana, konstitucionalna rane psihosomatske razgradnje, manjkavost instinkta samoodrzanja,

Sekundarna izmenjen odnos dete- majka I dete-okolina,

Prognoza uporne I dugotrajne, manje su ucvrscene I slozene, prolaze ili spontano ili na lecenje. 64.Licnost majke anoreksicnih devojcica? Poviseno bojazljiva, Agresivna, lako se osujecuje, Odgovara novim povecanjem agresivnosti, Preterano popustljiva majka biva jios popustljivija I trazi nacine da umilostivi dete da jede, Majka koja ne prihvata materinstvo, Ako je I sama anoreksicna, Depresivna, Detinjasta, Nestrpljiva, Kruta. 65.Projektivna identifikacija, ciljevi, Bion majka kontejner? Melkani Klajn opisujuci depresivnu poziciju, dolazi do zakljucka da ona pociva na shvatanjima o fantazmima, projekciji, pounutrenju, cepanju I zavisti. Zahvaljujuci cepanju u Ja veliki deo mrznje prema delovima vlastite licnosti upravljen je I projektovan Putem projekcije Ja se oslobadja losih delova. Objekat u koji se dete projektovalo lose delove postaje progonitelj koji sa svoje strane projektuje u dete ove lose sadrzaje. Objekt postaje predstavnik Ja. Bion , odojce projektuje u grupi, zatim u celu majku deo svojih sadrzaja. Ako je majka sposobna da “sanjari” sa svojim detetom, da prepozna sta se u njemu dogadja, sadrzaj koji dolazi iz odojceta podleze obradi. Majka postaje kontejner sadrzaja koje je dete u nju projektovalo I misli o njima. Ovako promisljen, izmenjen, ublazen I oslobodjen neprijateljski sadrzaj dete moze da pounutri ponovo. Ovo je put ka jacanju Ja I stvaranju aparata za misljenje deteta. Ciljevi projektivne identifikacije Da se otarase svog loseg dela,

Da poseduje object u koji je ubacio svoj deo, Da izbegne odvajanje od deteta, Intrusivna identifikacija- objekat se napada nasilnim prodiranjem u njega I svemocnim nadziranjem sto osiromasuje Ja. Projektivna identifikacija, dete izbacuje napolje deo svoje unutrasnje patnje, beta elementi. Osetljiva I domisljata majka zna da primi ono sto dete u nju projektuje, postaje kontejner tih sadrzaja I pretace ih u alfa elemente, obradjene I promisljene sadrzaje koje vraca detetu, koje ih ponovo pounutri.

66.Karakteristike Ja I Nad-Ja mladih sa mentalnom anoreksijom? Karakteristike Ja nedovoljno ustrojeno, slabasno, od unutrasnjih sukoba se brani cepanjem, izdvajanjem, nazadovanjem I projekcijama u sopstveno telo, Karakteristike Nad- Ja preuvelicano, ispunjeno osecanjem velicine I moci, povisena teznja ka samoljubivosti, samodopadljivosti, iskrivljena shvatanja o sebi podsticu neprekidnu potrebu za proveravanjem sebe I od strane drugih I postaju izvor neprekidnih osujecivanja, sukoba I kidanja odnosa sa drugima, 67.Razlika tuge od depresije, po Frojdu? Tuga, normalan odgovor na stvarni gubitak, nema samooptuzivanja, osoba sacuva samopostovanje, obavljuju se nova ulaganja u druge objekte, nema osiromasenja Ja, ni gubitka libido, Depresija, patolosko stanje, gubi se samopostovanje, dolazi do povlacenja libido sa objekta, izazvana dubokom I narcistickom povredom, siromastvo Ja, gubitak libido, nesposobnost za nova ulaganja, bira se objekat koji je slican osobi, koji predstavlja prosirenje njegovog selfa,

68.2 funkcije uzimanja hrane? Zadovoljstva koja proizilaze iz zadovoljenja bioloskih potreba za hranom, Zadovoljstva koja proizilkaze iz dodira sa dojkom, odnosno objektom preko sluzokoze usne supljine I koze oko usta, zadovoljstvo od uzimanja spoljnih objekata, igre usnicama, mljackanja, gutanja. Preko usta dete koje u prvim danima I nedeljama je nedovoljno pokretno uspostavlja prve odnose sa spoljnim svetom I na taj naciin ga ispituje,

69.Vreme javljanja neuroza, po Melani Klajn? Delovi licnosti I mehanizmi odbrane su veoma ranog porekla, iz prvih meseci zivota I vec na tom uzrastu postoji unutrasnji sukob, ( kako u okviru Ono izmedju nagona zivota I smrti tako I izmedkju Ono, Ja, Nad- Ja ), koji je gotovo nezavistan od okoline.

70.Uzroci hiperkinetickog sindroma? Uzroci mogu biti psihicki, koji deluju u okviru porodice I skole, organski, neurofizioloski, geneticki I drugi.

71.Kozna osetljivost? Mokrenje u postelju I rublje moze biti izraz teznji ka pasivnosti I za koznim zadovoljenjem kod mlade dece, 72.Koja deca nisu pogodna za psihoterapiju? Nisu formiran instance Ja I Nad- Ja, Poremecaji ponasanja, hiperkineticki sindrom, Umna zaostalost, sa psihoticnom depresijom, Depresivna, mutizam,

73.Odbacena I preterano zasticena deca? Preterano su usmereni na sebe, Imaju teskoce u odnosima sa drugima,

Pasivni su, Bez preduzimljivosti, Imaju osecaj manje vrednosti, Zavisni su,

Majke koprofaga? Prema Spicovim ispitivanjima one su nesvesno neprijateljske prema detetu I depresivne,

Konverzivni sindrom? Konverzivni sindrom je simbolicki izraz nesvesnog mentalnog sukoba. Nesvesne I potisnute instinktivne zelje prete da dopru do sveti. Konverzivni simptomi predstavljaju odbrane od instiktivnih zelja od kojih se Ja brani. Sadrzaji sukoba su potisnute libidne teznje koje su se razvijale do Edipne faze. Histericki sukob ima poreklo u falusnoj fazi,

74.Koji stadijum psihoanalize odgovara objektnom stadijumu kod Spica? Oralni stadijum,

75.Neuroza sukoba je, po Ani Frojd u Edipalnoj, a po Melani Klajn u Pre- edipalnoj fazi,

76.Koji mehanizam deca najcesce koriste u svojim igricama? Ponavljanje, 77.U koju vrstu poremecaja spade neosetljivost koze I sluzokoze? Poremecaj hranjenja, konverzivno sennzitivni poremecaj,

78.Preobjektivni stadijum, kako se naziva po Frojdu? Stadijum primarnog narcizma,

79.Autizam gde je ocuvana inteligencija, koji je to sindrom? Kannerov sindrom,

80.Na kom nivou psihoseksualnog razvoja se javlja fiksacija kod konverzija? Falusni nivo,

81.Razlike izmedju mehanizama odbrane I strategija prevladavanja? Mehanizmi odbrane su crta licnosti, po bihejvioristima, predstavljaju teorijski concept, spoznaju se posredno preko ponasanja, Strategije preovladavanja su bihejvioralne I izaziva ih stress, svesne su, predstavljaju nacine suocavanja sa konfliktima, product su zivotnog iskustva, ucimo kako da se suocimo sa problemima, nekada idu zajedno sa mehanizmima odbrane,

82.Poremecaj sa najgorom prognozom? Demencija,

83.U kom period su fiksirane osobe sa granicnim stanjem? U ranom uzrastu,

84.Da li adolescenti koriste projektivnu identifikaciju? Da stvaraju idole, 85.Analaizom perioda separacije, individuacije bavila se? Margaret Maler, 16 meseci do 3 godine,

86.Koji odbrambeni mehanizam je kada se stalno ponavljaju price deci? Repeticija, prisila ponavljanja,

87.Sta su DSM I DM? DSM- dijagnosticko statisticka merila, DM- dijagnosticka merila,

88.Najpoznatije Hartmanovo delo? “ Ego psihologija I problem adaptacije “

89.Navedite bar jednog autora koji se zalagao za concept strategije prevladavanja? Lazarus I Mus,

90.Autisticna deca sa vecim IQ- om? Karner, IQ autisticne dece je najcesce normalna, a niza je ravan intelektualnog funkcionisanja, pa je nizi IQ posledica poremecenog odnosa sa svetom. 1/3 dece nema ostecen IQ,

91.Deciji crtez? Ana Frojd se ne zadovoljava jedino verbalnim materijalom I takodje se obraca crtezima koje dete pravi u toku seansi. Cesto se dogadja da crtezi zamenjuju svaki drugi nacin komunikacije izmedju deteta I terapeuta. Dete se moze obratiti crtezu da bi na svoj nacin predstavilo stvarne scene ili imaginarne. Price koje osim latentnog znacenja koje se ocrtava iza ovih jasnih produkcija, moci ce da sluzi kao mamac slobodnim asocijacijama,

92.Da li se nagoni svesno opazaju? Nagoni nemaju kvalitet da bi bili opazeni od svesti. Tek predstava o nagonu, vidna slika, slusni utisak mogu postati svesni. Najpotpunije osvescenje se postize kroz reci. Mentalni sadrzaj kojii

je povezan sa govornim ili culnim pamcenjem dobija potrebna svojstva da bi mogao dopreti do svesti. Drugi uslov da misao dopre do svesti je da bude katektirana,

93.Psihoanaliticki stadijum pretece? Stadijum primarnog narcizma, objekta, preobjektni stadijum Preobjektni stadijum, po psihoanalizi stadijum primarnog narcizma. Novorodjence zivi u svetu bez objekta, ne razlikuje se od ne Ja, grudi majke dozivljava kao deo sebe. Prelazi iz stanja zadovoljstva u stanje nezadovoljstva I obratno. Krajem drugog meseca ljudi zauzimaju neko odredjeno mesto u okolini odojceta. Beba odasilje znake “ poruke ” koji postizu svoj cilj. Ono ne opaza object nego dozivljava rasterecenje od neprijatnosti kad se odgovori na poruku. To je uslov da se prepozna object I uspostavi odredjeni stadijum. Stadijum pretece objekta, anaklikticki po psihoanalizi, od 8 do 12 nedelja. Opaza lice majke gledano spreda, sastoji se od cela, ociju I nosa. Prepoznaje ga samo spreda. 3 mesec, prati kretanja I smesi se. Uci prvi signal oko 6 nedelje tragovima, secanju. Zato se govori o pretecama objektu, koji povezuje lice majke sa dodirom preko usta I zadovoljenjem potreba tokom mazenja. To void dete od primarnog narcizma do objektnog stadijuma. Ovo je pored drustvenog osmeha tacka organizator, to je trenutak izlazenja iz odnosa sa sobom u odnos sa okolinom I prelomna je tacka u organizovanju buduceg razvoja na visoj ravni,

94.Navedite najmanje 3 teorije o uzrocima I nastanku tikova? I teorija o organskom poreklu tikova Komplikovane trudnoce, porodjaji, oboljenja koja su mogla da ugroze CNS, Organske neuroloske promene, II teorija o psihickom poreklu Korbett – simptomi medju tikerima, povisena agresivnost, strepnja, neposlusnost, nastupi besa, poremecaji spavanja, osecajne smetnje, porodicni odnosi, Majke, napete, zaplasene, preterano popustljive, nasilne, Psihodinamicko ucenje, tik- izraz potisnutog sukoba koji je nastao u porodicnoj sredini, III teorija o psihomotornom poreklu

Deca “ tikeri”, smanjena mogucnost da nadziru neuromisicni rad I zbog toga ispoljene agresivne teznje, nalaze nacin, narocito kod decaka, da se ulazu u misice, IV ostale teorije Teorija komunikacije, tik – je dijalog deteta I roditelja u kome je dete spreceno da ispolji svoju agresivnost,

95.Sa kojim poremecajima se javlja tik? Diskinezije, Stereotipije, Grcevi misica, Epilepsija, Prisilne neuroze, Hiperkineticki sindrom, Usporen razvoj pokretnosti,

96.Somnabulizam, uzrok I posledica? Posledica je sukoba izmedju zelja i odbrana Ja I Nad- Ja sistema. Tokom spavanja, cesce u detenjistvu nego odraslom dobu. Kod dece su sukobi “ mladji “ I posledica su izmenjenih objektnih odnosa, nezadovoljenih potreba za ljubavlju, sigurnoscu, polnih I agresivnih teznji , Prognoza- u najvecm broju slucajeva procesi sazrevanja I razvoja uzimaju prevlast I mnogi sukobi se resavaju drugim I primerenijim mehanizmima abreagovanjem I drugim nacinima, pa ne dolazi do organizovanja neuroza. U slucaju slabije integracije Ja u odredjenim nepovoljnim okolnostima omladinac, pa i odrasli odgovore na nacin slican onom u detinjstvu, iako se jos ne moze govoriti o neurozi. Pod nepovoljnim okolnostima, konverzivno- histericni odgovori se ustaljuju, njihov osnovni obrazac I jezgro ucvrscuju, pa se tako organizovana preneuroza ranije ili kasnije preobraca u konverzivno histericnu neurozu.

Licnost Plasljiva,

Nesigurna, Depresivna, Veca osecajna reaktivnost.

Prognoza Obicno dolazi do prestanka somnabulizma u pubertetu, uz manju psiholosko, psihijatrijsku pomoc. Jaci napadi somnabulizma koji traju dugo I koji se cesto javljaju mogu biti otporni na medikamentno lecenje I na psihoterapiju imaju goru prognozu,

97.Izbor konverzivnog organa? Izbor organa vrsi Ja odnosno njegove odbrane I primarni procesi misljenja, zavisno od prethodnog iskustva sa pojedinim delovima tela I energije ulozene u organ. Kod sasvim malog deteta , zbog slabosti Ja , izbor nije tacno odredjen I Trajan. Telo u celini postaje organ izbora I nacin saopstavanja,

98.Licnost histericne dece I omladine? Povisena detinjatost, Podvodljivost, Samoljubivost, Sebicnost, spremnost ka nazadovanju, Sklonost ka mastanju, Dnevnom sanjarenju, Uzbudjivanju, Teatralnosti, Laganju, Mitomaniji I klevetanju, Zelja da se bude zapazen I da mu se dive, Osetljiv na osujecenja, gladna osecanja, Tesko podnosi dugotrajne napore, Normalne inteligencije,

99.Koji su osnovni mehanizmi odbrane prisutni kod nastanka konverzija? Premestanje,

Sazimanje, Simbolizacija, Projekcija, U konverziji dete- omladinac svoje fantazije projektuje uglavnom u svoje telo, kao da postoji samo razlika izmedju unutrasnje I telesne stvarnosti,

100.U cemu je razlika izmedju konverzivnih I disocijativnih reakcija I stanja?

Disocijativni odgovori- izrazavaju napor Ja da se idvoji I udalji od nekih sadrzaja koji izazivaju strepnju od koje Ja ne moze da se brani na drugi nacin. Po ovome su bliski prisilnim odgovorima,

Od konverzivnih odgovora se razlikuju po tome sto dete- omladinac ne pokazuje javno svoj sukob preko tela, vec zeli da ga izbegne izdvajajuci ga I od okoline I od sebe samog.

Ova razlika nije uvek jasna, jer neke osobe , koje odgovaraju na ovaj nacin , traze prisustvo drugih, I javno , u maskiranom obliku, iznose svoj sukob I svoje zelje,

101.Koji su kriterijumi razlikovanja konverzivnih od psihosomatskih poremecaja? Ako poremecaj unutrasnjih organa nema simbolicko znacenje , nego predstavlja neposredno rasterecenje , to se ne moze shvatiti kao konverzivni odgovor, nego kao odgovor na strepnju ili kao psihosomatski poremecaj. Razlika nije uvek uocljiva. Oba poremecaja nalaze telesni nacin izrazavanja I donose sekundarnu korist. Konverzija je izraz unutrasnjeg sukoba I bez telesne promene, a psihosomatska oboljenja neposredno telesno izrazavanje afekata ili potisnute teznje sa telesnim promenama,

102. U kojoj fazi psihoseksualnog razvoja nastaju konverzivni poremecaji? Frojd- u falusnoj fazi,

103. Koji konverzivni poremecaji poticu iz pregenitalne faze fiksacije? -

Poremecaji glasovnog aparata, afonija, praceni “ oduzetoscu” usana I jezika,

-

Smetnje disanja,

-

Stucanje,

-

Povracanje,

-

Smetnje gutanja, disfagije,

-

Smetnje spavanja,

-

Smetnje mokrenja, polakiurije,

-

Anoreksija I bulimija mogu biti I konverzivnog porekla,

104. Konverzivni - senzitivni poremecaj? - Gubitak osetljivosti, - Bolovi I preterana osetljivost pojedinih delova koze I sluzokoze, bez odgovarajuceg organskog nalaza, javljaju se cesce kod omladine, - Kod dece su veoma retki , jer mladje dete tesko doseca I zamislja da postoje I smetnje osetljivosti. Vidjaju se u obliku “ gubitka osetljivosti “ koze ruku. - Neki omladinci gase cigarette na rukama, bockaju iglama I pri tom ispoljavaju upadljivu neosetljivost. - Mehanizam nastajanja konverzivne neosetljivosti udruzuje se I ispreplice sa mazohistickim I drugim teznjama, samopovredjivanjem I ispoljavanjima. - Opisane su neosetljivosti u obliku rukavica ili kratkih carapa, polovine tela, hemianestezije,

105. Na kom uzrastu mozemo govoriti o prisilnim neurozama? Rano, posebno oko 3 godine zivota se javljaju brojni obredi uspavljivanja, kao sto su klacenje, vrcenje cuperka kose ili uha prstima, sisanje jezika I usnica, uzimanje predmeta, igracke u postelju… U fazi mirovanja, kada se pojavljuju I namecu mnoga ogranicenja I zahtevi porodice, sireg drustva I skole, javljaju se I organizuju novi obredi, prisile I kompulzije

Pokreti rukama, Skakutanje, Brojanje, Pevanje, Ponavljanje reci. U pubertetu I mladosti ispoljavanja su sve slozenija I slicnija tome sto se vidja u odraslom dobu.

106. Dozivljaj prascene? Fantazam koji potice od Frojda. Predstavlja dramatizaciju edipovske situacije u kojoj je Frojd prepoznao pocetak pravog secanja. Sporno je da li postoji prvobitni fantazam o primarnoj prasceni bez prisustva polnom cinu izmedju roditelja, ili dete svoje misli o polnom sjedinjenju izmedju roditelja razvija nakon gledanja ovog odnosa. Dete moze polni odnos izmedju roditelja da shvati kao agresivni I sadisticki cin na osnovu prividnog izgleda “ sile “ koji ovde primenjuje jaci na slabijeg, otac nad majkom. Ovakva shvatanja mogu da budu pracena strahom od heteroseksualnih dodira u odraslom dobu.

107. Kakva je prognoza opsesivno- kompulsivnih neuroza? Razvoj prvih karakteristika ispoljavanja u detinjstvu je neizvestan. Pod povoljnim okolnostima mnoga od njih su prolazna, ucestvuju u stvaranju I ulaze, u sastav Ja, koje poprima manje/ vise prisilna obelezja, ali u okvirima normalnosti. Prolazni obredi ne remete funkcije Ja i ostavljaju dovoljno mogucnosti za nove sublimacije. Pod nepovoljnim okolnostima stvara se prisilni karakter sa naglaseno prisilnim obelezjima do krajnje neobicnih razmera. Redje se javlja I razvija prisilno stanje koje se sa povremenim promenama u nacinu ispoljavanja, poboljsanjima I pogorsanjima moze organizovati u pravu prisilnu neurozu vec u detinjstvu I zadrzati do odraslog doba.

Kad svi nesvesni mehanizmi odbrane, kao sto su izdvajanje, potiskivanje, premestanje, intelektualizacija, racionalizacija I reaktivna formacija otkazu, postoji opasnost da nazadovanje dosegne psihoticnu ravan.

108. Simbioticka psihoza? Osnovna obelezja Jaka strepnja I panika na odvajanje od znacajnih, detetu bliskih osoba, najcesce majke. Naglasena dvojnost odnosa sa bliskim osobama. Dvojnost proizilazi iz snazne zelje za sjedinjenjem sa dobrim delovima objekta I istovremene teznje da se izbegnu losi delovi. Moze da pocne rano vec krajem prve godine, ali najcesce se javlja izmedju 3 – 6 godine, u edipalnoj fazi, kod dece koja su se pre toga razvijala neupadljivo , ili su bila upadljivo razdrazljiva. Bolest obicno pocinje nakon neke, maker I beznacajne povrede , odvajanja od bliskih osoba, medicinskih postupaka. 109. Mehanizmi odbrane u prepsihozama I prognoza I neuroze? Prepsihozama kao I neurozama svojstvena je raznorodnost I neuskladjenost Ja I njegovih odbrana. Za razliku od neuroticnih , prepsihoticne odbrane primitivnije su I ponavljaju se. To su Projektivna identifikacija, Pounutrenje, Premestanje, Poricanje, Cepanje, Strepnja , cesce strepnja od unistenja , a redje od kastracije , kod prepsihoza , je dublja , preplavljuje funkcije Ja , preti njegovoj celovitosti I povezanosti , pa su nova ulaganja otezana , sto se ispoljava kroz smanjenje novih objektnih veza. Intelektualne sposobnosti prepsihoticnog mogu biti dobre I pored slabog intelektualnog funkcionisanja I neuspeha u skolki.

Prepsihoticna deca ispoljavaju veliku spremnost da neprimetno odgovaraju na uobicajene okolnosti suporostavljanjem , povisenom agresivnoscu I prelaskom na cin. Oko 2/3 rano psihoticne dece ostaje ometeno u drustvenom zivotu I nesposobno za samostalan zivot. Postoji mogucnost da neka prepsihoticna deca kasnije ispoljavaju prividno neuroticne klinicke slike. Iako dete uspostavi nekakvu ravnotezu , postoji mogucnost da se pod novim stresovima I sukobima, pri pokusaju uspostavljanja novih objektnih odnosa I zbog teskoca novih ulaganja , ova ravnoteza poremeti , tad a dodje do novih povremenih , ali I trajne psihoticne razgradnje. Ako simptomi I odbrane ne uspevaju da nadziru duboku strepnju I depresivnost, ako su kocenja jaka pa suzavaju granice Ja, ako Ja nije u stanju da koristi nagonsku energiju prognoza je losa. Uzroci Organski- ucestalost porodjajnih I posleporodjajnih oboljenja , oboljenja misica , poremecaj paznje. Biohemijski- kateholamini. Geneticki- unutrasnje matericne nepodudarnosti izmedju ploda I majke – rani autizam. Znacajno je za sizofreniju I manicno – depresivnu psihozu. Psihicki I drustveno kulturni- uzajamno delovanje izmedju licnosti roditelja I posebnih obelezja deteta.

110. Intelektualizacija po Ani Frojd I cilj? Ana F. primecuje das u mladalackom dobu svojstveni intelektualizacija I asketizam kao nacini odbrana od naraslih nagonskih teznji I straha. Intelektualizacija svedoci o pokusaju da se upravlja naraslim nagonskim teznjama vezujuci ih za, I omladincu I drustvu, prihvatljive misli.

111. Kada je indikovana psihoterapija kod hiperkinetickog sindroma?

Indikovana je kada se sumnja na postojanje unutrasnjeg konflikta koji lezi u osnovi poremecaja. Cilj intelektualizacije je da cvrsto poveze instinktivne procese sa ideacionim sadrzajima I tako ih ucini dostupnima svesti I podloznima kontroli.

112. Normalno I patolosko kod dece I mladih u poremecaju ponasanja? Potrebno je razlikovati stvaralacko prilagodjavanje od nestvaralackog, odnosno od potcinjavanja. Zato dobro ili lose prilagodjavanje mora biti odredjeno kako u odnosu na prilagodjavanje prema sebi- autoplasticno, tako I u odnosu na druge- aloplasticno, da bi se procenilo njegovo pojedinacno psiholosko I drustveno znacenje I vrednost.

113. Koje su odlike asocijalnog tipa? Osecajne promene Pojacana strepnja I drugi oblici strahovanja, Pojacano neprijateljstvo, Mrznja, Ljubomora, Zavist, Depresija, Nesigurnost, Bes, Ljutnja, Tuga, Mrzovoljnost, Osecanje manje vrednosti… Poremecaji obrazaca ponasanja Durenje, Suprostavljanje, Svadljivost, Bekstva, Laganja, Neposlusnost, Tvrdoglavost, Naglost, Tuce…

Smetnj saznajnih funkcija Slabija paznja, Gubitak zanimanja za ucenje, Intelektualna zakocenost, Inhibicija, Mucanje… Preterani ili zakoceni pokreti Smetnje spavanja, hranjenja, mokrenja, defeciranja…

114. Projektivna identifikacija kod adolescenata?

Postoji, kod psihoticnih cija je licnost prepunjena zaviscu I pohlepom. Kakos u strepnja I neprijateljstvo jaki, projektovani delovi se cepaju na mnogo komada. Projektovanje ovih komadica u spoljasnji object stvara utisak das u I objekti koji ih primaju raspadnuti. Objekt je sad ispunjen neprijateljskim delovima. Svet se dozivljava kao da je nastanjen zastrasujucim I “ neobicnim “ bizarnim objektima. Delici licnosti I objekti se ujedinjuju u uzajamno – razornu vezu – znacaj u nastanku psihoza I border – lajna.

115. Kriza identiteta? Je prekid u do tada izgradjenom telesnom, polnom, osecajnom I drustvenom identitetu. Sastavni je deo mladalacke krize. Mlada osoba se ponasa kao da ne zna ko je I sta je, da li je ona ona ista ili neka nova. Prekidaju se poistovecivanja sa drugima. Usamljenost, Povisena narcisticka ulaganja, Postavljanje pitanja o sebi, Pocinje u pubertetu ili nakon njega u ranijoj, redje u kasnijoj mladosti, do 25 godine. Tok zavisi od prethodnog razvoja sklopova licnosti, odnosa izmedju stepena unutrasnje povezanosti I stavova okoline. Mogu predstavljati plodnu fazu I biti moguci “ organizator “ daljeg razvoja.

116. Koje su najcesce potrebe u pozadini decijih kradja?

Kod mladje dece sam cin prisvajanja moze da zadovolji potrebe za posedovanjem I da ima funkciju rasterecenja od napetosti I strepnje, funkciju trazenja kazne, pribavljanje ljubavi od roditelja, nadiknada za odbacivanje, suparnicenje sa povlascenim detetom. Ukradeni object moze da bude koristan, ali cesto ima I simbolicko znacenje, ljubav roditelja, prestiz naddrugom decom… Malo dete moze da veruje da je ukradeni object deo voljene osobe. Predmet se ili koristi ili poseduje, ali ga dete cesto I razara, krije I poklanja. U fazi mirovanja, narocito kasnije u mladosti, kradja je sve redje izraz neprilagodjenosti, a sve cesce izraz psihopatskih obelezja licnosti.

117. Paljenje kao poremecaj? Plamen deluje uzbudjujuce na culo vida I kozu. Svaka igra vatrom I paljenje ne mogu biti uvrstene u nedrustveno ponasanje. Jave se I kao normalna igra u detinjstvu, koja podstice uzbudjenja I mastu. Kao agresivna radnja. Prkos I otpor roditeljima I skoli. Nacin kaznjavanja drugih. Prisilni simptom u okviru prisilne neuroze ili prisilnog karaktera. Za starije decake vatra I paljenje- polno znacenje. Vatra I paljenje imaju I simbolicko znacenje, zezu, kaznjavaju, razaraju, ciste dobro od zla. Roditelji piromana, odbacujuci,osujecujuci, podsticu razaracke radnje.

118. Nabroj uzroke psihopatskih oboljenja? R. Spic- vec u 2 godini zivota. Uzrok grubo izmenjeni objektni odnosi izmedju deteta I roditelja.

Deca su u porodici izlozena jakim I dugotrajnim osujecenjima, odbacivanju, zanemarivanju, neprijateljstvu, grubom kaznjavanju, nedoslednim promenljivim I nepredvidivim stavovima roditelja. Neka deca su odrasla u preterano popustljivim porodicama bez ikakvih ogranicenja ili prepustena sama sebi I ulicnim grupama. Neki autori govore o psihopatijama pretezno neuroticnog tipa. Tada su radnje pracene naknadnim osecanjem krivice, iako je osecanje krivice za vreme radnje prividno odsutno. Prvobitna I naknadna agresija roditelja, Konstitucionalni, Organski, Temperament, Drustveno – kulturni.

119. Sta je mirecizam? Prezivanje hrane, funkcionalni poremecaj vracanja progutane hrane u usta, prezvakavanja I ponovnog gutanja. Nije cesta pojava. Javlja se kada je dete ostavljeno samo sebi. Udruzeno je sa kompulzivnim autoerotskim radnjama, klacenjem glavom I sisanjem prsta. Posledica je odbacivanja od strane roditelja, agresivnosti I manjkavih odnosa sa objektima, kao I nedostatka drugih podrazaja.

120. Anksioznost po razlicitim stavovima? O. Rank- porodjajno poreklo. On smatra da je rodjenje prva opasna, dozivljena okolnost koja predstavlja povredu I jak stress za dete kako u fizioloskoj, tako I psihickoj ravni. S. Frojd- posledica neposrednog pretvaranja libido zbog osujecenja I potiskivanja polnog nagona, da bi kasnije, sasvim izmenio svoje shvatanje I izjavio da strepnju izaziva unutrasnja napetost, ( libidnog I agresivnog porekla ), I da zapravo strepnja pokrece potiskivanje. M. Klajn- sukobi izmedju nagona smrti I zivota koji se odvijaju u okviru Ono, a zatim sa paranoidno- sizoidnom pozicijom I projekcijom agresivnih teznji u bliske objekte, sto je izvor strepnje progonjeja I sa depresivnom pozicijom koja je izvor depresivne strepnje. Strepnja je za nju unutrasnjeg porekla. M.Maler- prekid sibiotskog odnosa izmedju deteta I majke I dozivljavanjem nastajanja, a zatim procesima odvajanja I individuacije koji se odvijaju u 3 faze.

H.S.Salivan- smatra da bojazljiva I nesigurna majka neposredno, putem empatije, pogleda, glasa prenosi strepnju na svoje dete.

121. Teorije nastanka anksioznosti? Teorija ucenja Dete koristi strepnju da bi se povuklo iz normalnih drustvenih medjuodnosa I medjudelovanja. Zastrasujuca iskustva mogu izazvati strepnju. Kakos u iskustva strepnje neprijatna pojacavaju se izbegavajuci odgovori koji treba da smanjuju strepnju. Ovi neprilagodjeni odgovori menjaju ponasanje I drustvena medjudejstva. Egzistencijalisticka skola Strepnja nije nauceno iskustvo, vec je prirodno stanje organizma koje se javlja od pocetka zivota kao deo ljudskog postojanjka I podstrekac za resavanje osnovnih pitanja ccoveka. Istinsko ljudsko postojanje se suocava I podnosi egzistencijalnu strepnju. Kulturna psihijatrija Razlicito se objasnjava. Govori se o osnovnoj strepnji koju dete iskusi zbog osecajne nesigurnosti I neprijateljstva, koje dolazi iz spoljne sredine, ravnodusnosti, nemoralnih ponasanja odraslih, prevelike ili premale odgovornosti… ( Hornaj ). Psihoanaliza- posledica je iznenadnog preplavljivanja organizma I nezrelog CNS deteta na rodjenju, culnim I fizioloskim iskustvima.

122. Klinicki oblici ispoljavanja strepnje? - Povremeni napadi strepnje izazvani povredama, - Napadi strebnje bez vidljivog povoda, - Hronicna strepnja, - Strepnja od odvajanja.

123. Na kom uzrastu pocinjemo da govorimo o decijim fobijama umesto o strahu?

Najveci broj strahova kao I strepnja javljaju se normalno u raznim fazama zivota I polagano iscezavaju sa sazrevanjem, stvaranjem primerenijih odbrana Ja, I sticanjem stvarnih iskustava koja ispravljaju mastu I nepotpunu procenu stvarnosti. U tom slucaju se moze govoriti o normalnim fobicnim odgovorima ili stanjima. Tek kada se ovi fobicni odgovori uoblicavaju I organizuju, kad celokupna licnost deteta I omladinca poprima neuroticnu organizaciju sa svesnim sukobom u pozadini, nazadovanjem, teskocama u funkcionisanju I ostalim simptomima, moze se govoriti o organizovanim fobijama. Ove se ne vidjaju u ranim mesecima I godinama zivota, ali se njihovi prapoceci mogu nazirati vec u edipnoj I fazi mirovanja.

124. Samopovredjivanje sta predstavlja I uloga? Pojava koja se tokom razvoja vidja kod svakog deteta grize se, grebe, bocka, cese, cupa kosu… U prvim godinama moze da predstavlja refleksni odgovor na drazi. Nacin rasterecenja I oslobadjanja od napetosti I strepnje. Nacin ispitivanja svog tela, stvaranje telesne seme I telesnog , a zatim psihickog Ja. Nacin prilagodjavanja okolini. Oko 4 godine I kasnije polagano iscezava. Starija deca I omladinci bezazleno, ali I ozbiljno ( gasenje opusaka na rukama, povredjivanje ocnih jabucica, rezanje vena…) Obavlja se stalno ili u nastupima. Omladinac se stalno povredjuje mirno I ocekuje odgovore okoline ( koja je obicno iznenadjena I zaplasena ). Uloga- rasterecenje od osecajne napetosti upravljanje svojom okolinom.

125. Fobicne opsesije? Unjima se javljaju protumere cija je funkcija zastita od fobicnih dozivljavanja. Nije uvek jasna granica izmedju fobicnog I prisilnog sindroma, ali je jasna u starijem uzrastu. Kad je fobicnost veoma jaka I kad zataje odbrane kao sto su premestanje, potiskivanje I izdvajanje, a strepnja I fobija preplavljuju Ja, koriste se mnoge radnje koje poprimaju kompulzivna svojstva sa ciljem odbrane od nedozvoljenih teznji I nadziranja fobogenih osoba, predmeta I okolnosti. Protivmere- paljenje svetla kod fobije od mraka, mirisanje hrane kod fobije od hrane ili prisilno bavljenje sportom, mogu da otkriju poreklo potisnutog sukoba. Ove protumere nikada ne uspevaju potpuno I trajno da odstrane napetost I strahovanja.

126. Poremecaji psihoseksualnog identiteta u detinjstvu I adolescenciji? Pojacana polnost, Smanjena polnost, Promene u izboru objekata zadovoljenja ( polno samozadovoljavanje, homoseksualnost, fetisizam, zoofilija), Promene u nacinu nagonskog zadovoljenja, voajerizam, ekshibicionizam, sadomazohizam, transvestitizam, transseksualizam, Ostali poremecaji, rani pubertet, polna nezrelost, urodjeni nedostaci… 1- 4 – uzroci pretezno psihicki I drustveno- kulturni. 5- neuro- hormonski.

127. Objasni pojam preobjekta? Objekt predstavlja osobu, deo tela, predmet I sve ono sto skluzi ulaganju nagonske energije I zadovoljenju nagonskog cilja. Moze biti spoljasnji, unutrasnji I deo vlastitog tela. Beba ne razlikuje objekte, ni majku, mada ona odgovara na njene signale. Ona odasilje znake koji postizu svoj cilj. Ona ne opaza object, ali dozivljava rasterecenje od neprijatnosti, od gladi onda kada je odgovoreno na njen znak ( plac ).To je uslov da se prepozna objekt I uspostavi odnos sa objektom. 128. Psihodinamika I uzroci pojacane polnosti? Uzroci Dublji sukobi u odnosima sa porodicom, Osecajna glad, Depresivnost, Kompulzivnost, Strepnja, Suprostavljanje, Manjkavost u razvoju Ja I Nad- Ja sistema, Smanjivanje zabrana od roditelja I drustva. Vidjaju se rano kod izvesnog broja mladih u svim slojevima drustva. Decaci I devojcice rano stupaju u polne odnose. Cesto menjaju partnere. Devojcice ostaju rano trudne I postaju majke. Svesna, nesvesna erotska ponasanja roditelja ( promiskuitet, prostitucija ) I podvbodjenje, podsticu oponasanja I poistovecivanja I razvoj polnih zelja I teznji, pa I ispoljavanja.

129. Kako F. Dambar tumaci psihosomatske poremecaje? Uocavaju se zajednicke psiholoske osobine psihosomatske licnosti bez obzira na vrstu poremecaja. Prva osobina- nesposobnost bolesnika da uoce psihogenu pozadinu svojih telesnih zbivanja. Druga- snizen kapacitet simbolickog izrazavanja. Skloni su tehnickom resavanju problema uz smanjenu sposobnost za umetnicki dozivljaj. Imaju potrebu za redom, radom I tacnoscu. Simbiotski zivotni stil. Simbioze su cesto ambivalentne, ali ih oni prekinu, moguce su psihoticne dezintegracije sizofrenog tipa, depresivnog ili manicnog, toksiomanije ili suicidnost. Svoje konflikte izrazavaju u okviru socijalno dozvoljenog, jer samim prihvacanjem uloge bolesnika kroz psihosomatske simptome izbegavaju asocijalnost.

130. Stadijum ogledala? J. Lancan- “stadijum ogledala “ predstavlja vazan trenutak u stvaranju jedinstvene slike o sebi I u dozivljavanju sebe kao celine. Svoju sliku u ogledalu dete dozivljava I prepoznaje kao celinu koja vise nije “ onaj drugi “ . Dozivljavajuci svoje telo kao celinu, dete se oslobadja mucnih dozivljavanja svog tela kao iscepkanog, rasparcanog I stopljenog sa drugima, iako su drugi I te kako prisutnih u njemu I ostaju tamo do kraja zivota. Sve do 7 godine slika se pretezno odnosi na oblik tela ( figurativna ) , a zatim postaje slika tela koje dejstvuje ( operativna ), sto se moze videti I na decijem crtezu.

131. Nabrojite 3 organizatora psihe po Spicu? -

Znak odabirnog emocionalnog odgovora na podsticaj spolja ( pojava socijalnog osmeha ), Strah od odvajanja, Usvajanje I koriscenje semantickog znaka ne kao prihvacenih zahteva drustvene sredine.

Struktuira opazanje I postavlja osnove Ja,

Integrise objektne odnose sa nagonima I ustanovljava Ja kao psihicku strukturu sa razlicitim sistemima I funkcijama. Stvara put za razvoj objektnih odnosa uz pomoc semantickog opstenja.

132. Da li se razlikuju decaci I devojcice koji se umokravaju po osobinama licnosti? Devojcice pokazuju vise teznji ka nezavisnosti, takmicenju sa decacima, ambicioznije su. Decaci pasivniji, zavisniji ( narocito od majki ), manje agresivni, skloni povlacenju I samoobezvredjivanju. Decaci I devojcice koji se umokravaju su cesce plasljivi, imaju nocne strahove I kosmare, vezani su za roditelje, skloni nazadovanju, sisanju prsta, detinjasto ponasanje. 133. Simbolicka funkcija enureze kod devojcica? Devojcica se oseca uskracena zbog toga sto nema muski polni ud I cinom mokrenja porice svoj nedostatak. Za nju umokravanje predstavlja suparnicki I osvetnicki cin.

134. Funkcija umokravanja? Psihoanaliza- metodolosko znacenje Mokraca, voda I amnionska tecnost imaju zajednicka svojstva I umokriti se, biti vlazan, moze simbolicki da izrazava prvobitnu zelju za vracanjem u matericu I plivanjem u tecnosti majke. -

Nacin suprostavljanja okolini,

-

Uziva u prljanju, biti prljav je vlastiti cin deteta I njegova je volja, Teznja ka pasivnosti, Teznja ka koznim zadovoljstima koja obezbedjuje toplo I vlazno rublje, Depresivni odgovor ( plac kroz besiku ) na gubitak objekta ili odbacivanje, Izraz moci I snage, nadmoci nad okolinom, Suparnicki I osvetnicki cin, Zamena za polno samozadovoljavanje…

135. Poistovecenje sa agresorom? Predstavlja “ manevar “ izbegavanja strepnje na taj nacin sto se postaje slican zastrasujucem objektu, a agresivnost se dozivljava kao da potice iz samoga sebe I na taj nacin se moze kontrolisati. Da bi se zastitilo od osobe koja plasi I izbeglo strah od njene agresije, dete se poistovecuje sa agresorom I tako ponasa. “ Dobro “ I dosledno poistovecivanje sa losim likovima ima ulogu u pojavi nedrustvenih ponasanja. Ako se to desi od prvih dana nedrustveno ponasanje postaje jedini moguci nacin ponasanja.

136. Uzroci intelektualne inhibicije? Neuroticni sukob Intelektualne inhibicije neuroticnog porekla su aktivne. Kocenje intulektualnih delatnosti je funkcionalno, a cilj mu je da se izbegne neuroticna strepnja. Kod histericnih se ispoljava kao intelektualna slabost, smetnje pamcenja I nedostatak, secanja, odnosno kao smetnja ucenja I smetnja zadrzavanja naucenog. Pocetni psihoticni process Intelektualna inhibicija psihoticnog porekla je slozenija I pasivna, pored navedenih simptoma se javljaju I smetnje razumevanja, koncentracije paznje I siromastva intelektualnih stvaranja. Psihoticni ucenik nema uvid u svoju neuspesnost I sklon je da porice.

137. Izdvajanje, izolacija- gde se javlja? Pomocu njega osecanja se izdvajaju od misli I udaljuju od ostalih sadrzaja dusevnog zivota. Ovaj mehanizam snazno deluje u prisilnoj neurozi, fobiji, psihozi, disocijativnih oblika histerija, psihopatiji I autizmu.

Ovako otcepljena I izdvojena osecanja su potisnuta I dete- omladinac ne nalazi nikakvu vezu izmedju ova 2 sadrzaja. Tako se izbegavaju strepnja I osecanje krivice I sprecavaju, udarac, polni napad… U normalnom dusevnom zivotu osecanje, misao I radnja su u skladu I povezani. Kad deluje mehanizam izdvajanja misao postoji bez osecanja, a dete nije motivisano da ucini, ostvari svoju zamisao.

138. Koja je svrha intelektualizacije kod omladinaca? Intelektualizacija je bliska odvajanju. Zahvvaljujuci njoj omladinac, a I starije dete, uspeva da osecajne sukobe podvrgne pod sekundarne procese misljenja I govora I da ih na taj nacin obradi I ublazi strepnju koja bi proizislka iz sukoba. Ako su mehanizmi odbrane, kao sto su potiskivanje, izdvajanje, intelektualizacija, isuvise jaki, afekt gubi svoju funkciju prilagodjavanja I Ja ostaje ugrozeno.

139. Ekran secanja? Amnezija obuhvata sve bitne dogadjaje ranih konfliktnih situacija ako bi ovi, zbog afektivne nezrelosti deteta, bili suvise mucni da bi im dete pristupilo, suvise teski da bi bili reseni, te bi jedino ponistenje omogucilo detetu da ih zaboravi umesto da ih resi. Tako cele povrsine detetovog zivota propadaju u zaboravu, dozvoljavajuci cesto da prezive samo indiferentna secanja, ekran secanja, mnesticki predstavnici liseni svakog afekta.

140. Prvobitno I naknadno potiskivanje? Prvobitno Koristi se rano da se obuzdaju prvobitni izdanci Ono I pre njihovog dopiranja do praga svesnosti. Naknadno Koristi se sa ciljem potiskivanja sadrzaja koji su vec tokom razvoja bili u ravni svesnog I protiv uticaja I delovanja Nad- Ja. Frojd

Potiskivanje se odnosi na psihicke predstavnike pulzije I ovi psihicki predstavnici, neprihvatljivi za Ja, postaju odbaceni izvan svesnog u prvi stadijum koji je Frojd nazvao primarno potiskivanje. Ubuduce, svaki dogadjaj, svako uzbudjenje, svaka ideja koja je bila povezana sa potisnutim psihickim predstavnikom putem asocijacije, trpece sa svoje strane potiskivanje. Isto tako I nizovi ideja, koji proizilaze I koji su povezani sa predstavnikom pulzija, bice odbaceni iz svesnog procesom koji je nazvan sekundarno potiskivanje.

141. Adhezivna identifikacija? E. Bick uvodi pojam “ adhezivna identifikacija “ cije je obelezje zavisnost I potreba za slepljivanjem sa povrsinom objekta, kako bi se izgradila “ psihicka koza “. Adhezivna identifikacija prethodi pounutrenju delimicnog objekta.

142. 3 stavke projektivne identifikacije? Dete I dusevni bolesnik imaju fantazam svemoci I fantazam o tome da mogu deo sebe da ubace u spoljni svet sa trojakim ciljem Da se otarase svog loseg dela, Da poseduju objekt u koji su ubacili svoj deo, Da izbegnu odvajanje od objekta, jer su sada sjedinjeni sa njim. Odojce/ dusevni bolesnik na taj nacin vrsi pritisak na objekt da ovaj, nakon primanja projekcije, dozivi osecanja vezana za projekciju. Projektovani delovi mogu ponovo da budu pounutreni ako ih objekt u koga su bili projektovani I koji ih je primio preinaci I ako postanu pogodniji za dete, odnosno dusevnog bolesnika.

143. Frojd- Ja ? Ja se diferencira u psihickom aparatu u kontaktu sa spoljasnjom realnoscu. Njegovo funkcionisanje je upravljeno sekundarnim procesima. Njegova struktura je odredjena principom realnosti. Ja Kontrolise voljni modalitet ( funkcija konzervacije ), Opaza stimulus ( mnesticka funkcija ), Registruje secanja (mnesticka funkcija ), Prilagodjava se drazenjima I uzbudjenjima ( funkcija prilagodjavanja ), Izbegava suvise velike napetosti ( funkcija izbegavanja ),

Proucava obavestenja koja mu pristizu I tezi da izmeni sredinu stvarajuci je podobnijom da odgovori na njegove potrebe I da ispuni njegove zelje ( funkcija ucenja ). Mehanizam odbrane ( Frojd ), Histericna neuroza- potiskivanje, Opsesivna neuroza- izolacija I reaktivna formacija.

144. Anakliticki strah? U period izmedju 6 I 8, do 10 meseci dete ulazi u objektni stadijum. Objekt, majka/ neka druga osoba se sada dozivljava kao Celina. Majka postaje “ psihicka majka “, koja ispunjava dete radoscu I onda kad ono nema bioloskih potreba ( za hranom … ). U ovom period prekid odnosa sa bliskom I odnos sa stranom osobom ispunjava dete strepnjom, opisanu kao anakliticki strah osmog meseca. Pojava anaklitickog straha je dokaz da majka postaje posebna osoba jedna I jedina u tom razdoblju koja ima svoju posebnu funkciju I ne moze biti pomesana sa drugim osobama.

145. Engleska skola- tehnike interpretacije? Interpretacije su date u ranom stadijumu analize, cesto od prvih seansi. Interpretacije se odnose na nesvesni sadrzaj igara I crteza, ciljanju u njima edipovske teme, introjekcije I projekcije, sukob izmedju spoljasnjih I unutrasnjih objekata, zavisno od primitivne scene I strahovanja od kastracije I komadanja. Na kraju se insistira na potrebi sistematskog interpretiranja negativnog transfera bez cega nijedna analiza ne bi mogla da bude dovedena do kraja I rizikovala bi da utone u imaginarnu igru sa terapeutom.

stvarnoscu I koje predstavljaju delove Ja bez sukoba. Znacajan cinilac u povisenoj povredljivosti je nemogucnost da se odgovori na drazi I da se nadziru rasterecenja. 146.Aloplasticne

Sposobnost da utice na odgovore iz sredine. 146. Po Hartmanu koje funkcije ucestvuju u procesu prilagodjavanja? Hartman govori o “ prilagodljivosti “ deteta I o autonomnim funkcijama Ja koje postoje I pre iskustva sa Autoplasticne Funkcije pomocu kojih se prilagodjava sredini.

147. Karakteristike vulnerabilne dece? Nize energetske mogucnosti. Manje krepka I delatna deca. Manje zainteresovana I poverljiva. Odnos sa roditeljima je slabiji. Povisena povredljivost CNS- a koja je najveca u prva 3, a zatim do 6 meseca embrionog zivota, za vreme stvaranja neuroblasta, iz kojih se stvaraju neuroni. Niska culna reaktivnost. Odbrambeno podignut prag podrazljivosti. Slabija pokretljivost. Uzroci Urodjeno slabije funkcionisanje autonomnog sistema u stanjima stresa, Uticaji iz spoljasnje sredine koji remete obrasce licnog funkcionisanja.

148. Tezisne godine? U elektrokortikalnom sarevanju to su godine prva, treca, sesta, jedanaesta, u kojima se bioelektricne promene zavrsavaju. Ove etape odgovaraju razvojnim fazama, prelasku od malog deteta ka predskolskom ( 3- 6 godina ), skolskom ( 7 godina ), pocetku puberteta ( 11- 12 ).

149. Indikacije za roditeljsku psihoterapiju? Indikacije izmicu krutoj sistematizaciji. U takvoj perspektivi dve su mogucnosti Da se savetuje roditeljima, ili jednom od njih, da apocnu analiticku psihoterapiju koja bi tezila da razresi neuroticno jezgro ukidajuci nesvesne faktoire koji odrzavaju psihicki konflikt.

Da se zapocne psihoterapija podrske, uz nadanje da ce izvesne promene stavova okoncati osecajne interakcije koje uslovljavaju porodicne teskoce. Svaka psihoterapija tezi da postigne osecajno sazrevanje, u onom smislu u kome se ocekuje da isceznu infantilna ponasanja koja se odrazavaju kod roditelja kao posledica starih neresenih konflikata. Svaka psihoterapija, ma kako bila povrsna, uplice ceo terapijski odnos u spor I tezi da istakne ne samoobjektivnu realnost, nego I sve to sto je obuhvaceno u imaginarni splet koji joj daje boju I simbolicko znacenje koje joj se pridaje.

150. Simptomi impulzivne opsesije? Prisilni strah od besmislenih radnji ( izgovaranja skaradnih reci, ispustanja vetrova, mokrenja na javnim mestima…). Strah od izvrsene radnje. Strah od zadovoljenja pulzije I zelje. Javljaju se razliciti poremecaji ponasanja ( kleptomanija, bekstvo, napadi besa, ekshibicionizam…) I ovi poremecaji mogu da budu opsesivno- kompulsivno impulsivnog porekla.

151. Rano afektivno lisavanje? Potrebe odojceta -

Potrebe prvog detinjstva, potrebe sisanja, potreba za stalnoscu. Posledice ranog afektivnog lisavanja.

Jedan od bitnih uslova, koji obezbedjuje mentalno zdravlje osobe jeste uspostavljanje osecajnog, trajnog odnosa izmedju majke I deteta, odnosa koji daje sigurnost. Ovaj odnos je neophodan da bi se kontrolisali strah I krivica, koji ne treba da prevazidju normalne psiholoske odbrane. Kad takav odnos nedostaje u prvom detinjstvu, govori se o ranom osecajnom lisavanju.

152. Uzroci enureze I na osnovu cega se postavlja dijagnoza? Uzroci Osecajni cinioci,

Porodica, drustvena sredina I umokravanje, Drustvena sredina I mokrenje, Duboko spavanje I umokravanje, Potomanija I umokravanje, Ostali uzroci Neuravnotezenost neurovegetativnog sistema, Povisena napetost, Mali obim besike, Mentalna zaostalost, Promene CNS, Epilepsija. Postavljanje dijagnoze Laboratorijsko ispitivanje, Neurolosko ispitivanje, Metabolicko ispitivanje, Radiografsko ispitivanje, Bakteriolosko, Urolosko, Unarazvucno…

153. Indikacije za psihodinamsku psihologiju ili psihoterapiju? Potrebna je strukturna dijagnoza, koja treba da govori o neuroticnom trpljenju, bilo da osoba sama dolazi da trazi pomoc pod pritiskom neizdrzivog straha, bilo da karakterni ili funkcionalni symptom remeti njeno drustveno funkcionisanje, porodicni zivot, kapacitete za ljubav ili rad. Dijagnozu postavljamo na osnovu uporedjivanja izvesnog broja cinjenica dobijenih sledecim istrazivanjem Anamnesticko ispitivanje- odnosi se na biografiju I proslost deteta, iznosi se na videlonesklad libidnog razvoja, Klinicko ispitivanje- odredjuje se oblik I jacina poremecaja.

154. Kriterijumi normalnosti? -

Procena u odnosu na nazadovanje, U odnosu na onoga ko procenjuje I donosi sud, U odnosu na osobu koju procenjujemo, U odnosu na merila statistickog proseka, U odnosu na prilagodjenost I neprilagodjenost.

155. Normalno I patolosko u odnosu na prilagodjavanje? Prilagodjenost ( normalno ) znaci odsustvo sukoba, kako unutar sklopova licnosti , tako I izmedju osobe I drustva. Neprilagodjeno ( patolosko ) se obicno trazi u pojedinacnoj osobi, u njenoj uzoj drustvenoj sredini, a manje u sirem drustvu. Osoba se prilagodjava pod uslovom da -

Ostvaruje svoje urodjene sposobnosti, Delatno menja sebe I svest.

Prilagodjavanje po svaku cenu je patolosko.

156. Dva najglobalnija kriterijuma za razlikovanje normalnog- patoloskog? Patolosko bi predstavljalo nesposobnost mlade osobe da s obzirom na uzrast -

Zadovoljava svoje osnovne bioloske, osecajne, saznajne I drustvene potrebe. Da nadzire svoju agresivnost, depresivnost I strepnju. Da izdrzi osujecenja. Da se trajno ili barem najcesce ponasa na drustveno prihvatljiv nacin I ne ostecuje druge. Da ostvaruje pozitivne I stvarne drustvene ciljeve. Da se vpoistovecuje sa etickim vrednostima. Da ostvari svoje bioloske datosti. Da ostvari potrebnu ravan intelektualnog funkcionisanja.

Vidljiva jacina simptoma mora biti procenjena u odnosu na snage sistema licnosti da izdrze unutrasnji ili spoljni sukob koji je pojavu izazvao. Patolosko- poremecaj u nacinu organizovanja pojedinih sklopova, njihovih medjuodnosa I funkcija. -

Tendencija ka progresivnom razvoju. Kriterijum procene.

157. Mehanizmi odbrane indetifikacije sa agresorom I projektivne identifikacije?

Mehanizam odbrane projektivne identifikacije M. Klajn Uvela je projektivnu identifikaciju kao nov mehanizam odbrane. Pociva na njenim shvatanjima o fantazmima, projekciji, pounutrenju, cepanju, zavisti. Indetifikujuci se sa izvesnim delovima svoga Ja, projektovanih na spoljasnje objekte, ona omogucava osobi da se indetifikuje sa osecanjima spoljasnjih objekata. Mehanizam odbrane identifikacije sa agresorom A.Frojd Poistovecivanje sa mocnim, dobrim. Pounutrenje sa neprijateljskim I nedrustvenim porukama koje nesvesno upucuju roditelji.

158. Fiksacija falusne faze? Fiksacija ( zadrzavanje ) falusnih svojstava u odraslom dobu ometa put ka konacnoj gnitalizaciji I dalje mogucnost za razvoj neuroza, u prvom redu konverzivno- histericne neuroze.

159. Imitacija? Pored poistovecivanja koje pomaze jasnijem odvajanju sebe od drugih I sticanju identiteta, vrlo je bitna I imitacija ( oponasanje ) koje se vise ne svrstava u nesvesne mehanizme odbrane. Javlja se u 2 godini zivota.

160. Prednosti I nedostaci indetifikacije sa mocnim I neodoljivim roditeljima? Prednosti Vodi dete do osecanja vlastite moci, sto je uobicajeno I pozeljno. Nedostaci Prejako poistovecivanje kao zastita od gubitka svemocnog roditelja void dete ka pogresnoj slici u jedinstvu sa roditeljem I u strahu od svakog odvajanja, prekida odnosa, osamostaljivanja I stvaranja svoje osobenosti. Nazadovanje izazvano strepnjom odvajanja moze biti duboko I voditi ka povlacenju I autizmu, narocito mladjeg deteta, ili ce voditi do poricanja gubitka.

161. Najtezi poremecaji roditelja u odnosu na dete? Roditelji sa grubim karakternim promenama, narocito paranoidni, depresivni I plasljivi, kao I roditelji prividno ocuvanog spoljnog izgleda koji uvlace decu u izmenjena uzajamna delovanja I uticaje imaju jaci patogeni uticaj nego drugi tipovi dusevno bolesnih roditelja. Opasnost je utoliko veca ukoliko su oba roditelja bolesna. Ispitivanja su pokazala das u deca roditelja sa hronicnim telesnim oboljenjima sklona samoubistvu, kao I deca roditelja alkoholicara.

162. Zasto funkcionalni poremecaji nisu dovoljni da se dijagnostikuju? Funkcionalni poremecajii- reaktivni poremecaji ponasanja. Pojavljivanje fobicnih, opsesivnih simptoma kod deteta, skolskih teskoca, poremecaja porodicnog ponasanja, psihomotornih ispoljavanja nije dovoljno da se potvrdi neuroza ciju dijagnozu treba tacno dokazati. Uslovi sredine obicno iscezavaju kada se poprave odgovorni spoljasnji uzroci, dok osoba jos nije interiorizovala konflikt.

163. Cime se objasnjavaju poviseni izgledi za javljanje dusevnih smetnji kod dece roditelja dusevnih telesnih bolesnika? Uzajamnim uticajima, pounutrenjima I poistovecivanjem sa izmenjenim licnostima roditelja I delovanjem drugih mehanizama. Osecajnim osiromasenjem I povisenom agresivnoscu I bojazljivoscu bolesnih roditelja, narocito depresivnih I sizofrenih. Prisustvo dusevnog bolesnika u porodicnoj grupi. Odnosi izmedju dusevnog bolesnika I njegove porodicne grupe, s jedne strane, I drustvene sredine s druge strane. Prisustvo hronicnog telesnog bolesnika u porodicnoj grupi. Veci broj razvoda, napustanja porodice, prestupnistva I cesta bolnicka lecenja. Uticaj nasledja se pominje kao nedovoljno utvrdjen uzrok.

164. Usvojenje, kada saopstiti? Samo otkrivanje istine nema znacaja kao izdvojen cin I treba ga razmatrati u sklopu sveukupnih odnosa dete- roditelj. Oko 4 godine zivota dete pocinje da se zanima I postavlja pitanja o “ pravljenju “ I “ radjanju “ beba, pa I svom rodjenju. To je prilika da mu se otvoreno I bez dramatizacije saopsti das u ga rodili drugi roditelji koji nisu mogli da se brinu o njemu, ted a su usvojitelji roditelji koji su to preuzeli.

165. Kognitivna terapija kod depresije? Kognitivna restrukturacija ohrabruje decu da situaciju interpretiraju na drugi nacin. Dete ima drugaciji pogled, pa je situacija manje katastrofalna, manje znacajna ili cak pozitivna. Nadolazeca teskoca se sada posmatra kao izazov za uspeh, mogucnost za sticanje iskustva I prethodna iskustva se tumace u pozitivnom svetlu. Ovo je moguce I za mladju decu jer terapija predlaze novi okvir situacije, a dete treba samo da je prihvati.

166. Razlozi neuspeha u skoli? Treba novi odgovor.

167. Razvojna prognoza odstupanja I poremecaji? Odstupanja, usporenja ili ubrzanja, osecajnog, saznajnog I drustvenog razvoja koja su pretezno ili iskljucivo konstitucionalnog ili urodjenog porekla. Prognoza Iako su izazvana pretezno urodjenim ciniocima, to ne znaci da su konacna I das u moguca prinacavanja I otklanjanja bilo spontana ili uz strucnu pomoc. Poremecaji – Engel -

Odstupanja od opstih obrazaca sazrevanja. Odstupanja u posebnim oblastima razvoja razvoj pokretnosti, opazanje spoljasnjih drazi,

razvoj govora, saznajni razvoj, usporeni ili preuranjenmo razvijeni obrasci drustvenog ponasanja, prerano razvijena ili dugo odlozena zanimanja za polnost, nepostojanost ili usporen ili ubrzan razvoj sposobnosti za nadziranje osecanja strepnje, depresije, radosti, besa I ljutnje.

168. Navedite prirodu razvoja odstupanja, zasto ona ne moraju ukazivati na patologiju? Posledica su nedovoljno poznatih I neispitanih unutrasnjih cinilaca, ali olaksana ili podstaknuta spoljnim ciniocima. Usporen/ ubrzan razvoj nekesposobnosti moze da bude posledica uzajamnog delovanja snaznih konstitucionalnih sklonosti, ometenosti cula I velike osetljivosti na delovanje spoljnih uticaja koji urodjenu sklonost mogu ili podstaci ili ometati I otkloniti. Na slican nacin mogu da deluju prerano rodjenje, porodjajne povrtede I komplikacije, ometenost cula, telesna oboljenja, koji pojacavaju neku urodjenu sposobnost ili sklonost.

169. Nema ga celo pitanje

170. Sta je pripremna faza, primarni process I obelezja? Pripremna faza. A. Frojd predlaze da svakoj psihoanalizi deteta prethodi pripremna faza. Smisao ove faze je da se od samog pocetka sredi nepogodna situacija, da se dete motivise da zapocne I izdrzi terapijski odnos. U toku nje osoba sa terapeutom moze da uspostavi veze zasnovane na poverenju. Terapeut treba da se ucini detetu kao osoba koja se interesuje za njega, koja mu moze biti korisna I koja raspolaze izvesnom moci. Terapeut treba da postane neophodan detetu. Primarni process Prvobitni nacin nesvesne misaone delatnosti u kome psihicka energija slobodno, bezvremeno protice od jedne predstave do druge, premesta se I zgusnjava.

171. Sta je falusni primat?

Frojd je primetio da se u falusnoj fazi, gde se parcijalne pulzije pokoravaju investiranju genitalnih zona, jedino ceni muski genitalni organ I to isto tako kod decaka, kao I kod devojcica. To je nazvao falusni primat.

172. Sta je geneticki pristup I kakav je kod poremecaja dece? Hartman I Kris su definisali geneticko glediste I suprostavili ga dinamickom. Geneticki pristup ide za tim da rasvetli ponasanje pozivajuci se na istorijsko poreklo ili na anamnesticke podatke. Geneticko glediste dokazuje da je svaki psihicki fenomen podvrgnut zakonima kauzalnosti I da istoriju uzroka treba slediti do njihovog porekla, dalo je podstrek metodama posmatranja. U svojim radovima, geneticko gledistte je akcenat na odnosu majka- dete I isticu prirodu objektivnog odnosa I posebno ispituju uticaj koji su na dozivljeno I licnost deteta mogli vrsiti izvesni tipovi materinskog ponasanja, cak izvesni drustveni I kulturni uslovi.

173. Razlike izmedju klasicne psihoanalize I teorije objektnih odnosa? Sa pojmom objektni odnos analiticka psihologija je tezila da objasni interakciju osobe I okolne sredine- kako preko postiupnih faza razvoja osoba malo po malo uspostavlja svoje veze sa objektima , kako se odvija prelaz od fizioloskog ka psiholoskom, od matericnog parazitizma do ustanovljenja hijerarhizovanog drustvenog odnosa. Kod psihoanalize izraz objekt je ono sto obezbedjuje zadovoljenje pulzije. Po Frojdu objekat nije nuzno spoljasnji I moze da sacinjava deo vlastitog tela. Teorija objektnih odnosa- libidonozni objekat se moze promeniti u toku razvoja sledece faze sazrevanja pulzija, odredjen je trajnoscu svoje funkcije. Klasicna psihoanaliza- objekat moze biti odredjen na Trajan nacin svojom vremenskoprostornom koordinacijom.

174. Terapija zlostavljene dece? Podrazumeva kako terapiju roditelja, tako I terapiju deteta. Terapija koja ce se primeniti zavisice od vrste zlostavljanja –fizicko, emocionalno, sksualno, zanemarivanje.

Treba ispitati gde je dete zlostavljeno, u porodici ili van nje. Treba uvideti koji je clan porodice u pitanju. Ostali treba da pruze podrsku I zastitu detetu I umanje osecaj krivice. Terapija ohrabrivanja. Terapija igrom. Kognitivna terapija- trebalo bi da pomognu detetu da stekne uvid da nije krivo, vec da je greska osobe.

175. Roditelji dece sa konverzijama? Kod histerija? Roditelji koji snazno osujecuju bioloske, osecajne, saznajne I drustvene potrebe svoga deteta. Erotizovani histericni roditelji koji erotizuju odnose sa svojom decom u svim fazama razvoja. Mazohisticki roditelji.

176. Kastracioni strah kod decaka I kod devojcica? Decak Kastracioni strah mu omogucava izlaz iz edipalne situacije. Stapanje libidnih I reaktivnih teznji pod pretnjom kastacionog straha, odnosno pod uticajem stvaranja moralnih normi pretvara tako edipalnu privlacnost prema majci u sinovljevu ljubav. Devojcice Kastracioni strah dovodi do edipalnog kompleksa. Razresenje edipovog kompleksa je slozenije. Da bi ga resila, ona po drugi piut mora da napusti objekat ljubavi I zelje ( sada oca ) I da se vrati prvobitnom objektu ( majci ). Ovo je moguce ako prihvati cinjenicu anatomije sopstvenog pola I iskoristi sustinski sadrzaj kompleksa kastracije koji “ inhibrira I ogranicava muskost, ali obecava zenskost”. Postepeno napustanje edipovog kompleksa putem potiskivanja ce kod nje u velikoj meri da zavisi od naslucivanja o materinstvu, zenskosti…

177. Sta je konstitucionalno? Podrazumeva sve urodjene faktore koje dete donosi sa sobom na rodjenju. Oni obuhvataju kako nasledne, tako I stecene cinioce u toku intraliterinog zivota. Zavise od fizioloskog I psiholoskog stanja majke.

178. Razlike u shvatanju transferne neuroze po Ani f. I Melani Klajn? M. Klajn- konstatuje da simptomi variraju zavisno od ovijanja analize I od neuroze transfera. A. Frojd- nema “ prave “ neuroze transfera kod deteta u tojkku koje analiticar u afektivitetu malog pacijenta zamenjuje prvobitne objekte. Za nju je svakodnevna okolina deteta, narocito njegovi roditelji, ukljucena u polje lecenja. Za M. Klajn stvarna okolina zauzima samo podredjeni polozaj u vezi sa unutrasnjim subjektivnim konfliktom. Za analizu vrede jedino projekcije I introjekcije dobrih I losih objekata. Transfer ima svoje poreklo u procesima koji u najranijim stadijima odredjuju odnose s objektima.

179. Razlika izmedju neuroticnih I psihoticnih mehanizama odbrane po M. Klajn? Psihoticni mehanizmi odbrane, cepanje, napusteni I zamenjeni “ neuroticnim “ odbranama, potiskivanjem, premestanjem I inhibicijom. Ona smatra da paranoidne I depresivne pozicije predstavljaju tacke psihoticne fiksacije. Kad osoba regredira na ove ahaicne faze svoga razvoja, ona gubi znacenje realnosti I postaje psihoticna. Ako je osoba bila sposobna da prevazidje ove stadije, nece regredirati na taj nivo I moci ce postati neuroticNA. Ona vidi razliku izmedju psihoze I neuroze uglavnom u dubini regresije- neuroticna regresija ne dostize nikada prve faze. Psihoticni mehanizmi odbrane- cepanje. Neuroticni mehanizmi odbrane- potiskivanje, premestanje, inhibicija.

180. Psihoterapija za kontrolu besa? Socijalna- vezbanje socijalnih vestina. Psiholoska. Medikamentozna- neuroleptici, antidepresivi. Resavanje problema. Anger management. Igranje uloga.

181. Ne znam sta je sa tim pitanjem

182. Nemam pojma

183. Ana Frojd problem transfera kod psihoanalize deteta? Ona smatra neophodnim da dete uspostavi srdacne odnose sa terapeutom, “ pozitivan transfer “. Uvidja das u odnosi izmedju deteta I terapeuta afektivno nabijeni ljubavlju ili privrzenoscu, pobunom ili suprostavljanjem. Ona kaze da ova osecanja ne sacinjavaju pravu neurozu transfera. Ako u toku analize odrasla osoba zaista moze da dezinvestira objekte sa kojima je povezuju fantazmi I spremna je da ponovo stvori svoju nervozu u analitickoj situaciji gradeci je oko osobe analiticara, takav transfer izgleda nemoguc u sklucaju deteta it to iz 2 razloga Za dete, pocetna situacija- odnos sa roiteljima- ostaje uvek aktuelna, jer ono zavisi od njih. Ovi konfliktni objekti su spoljasnji I jos nisu interiorizovani kao kod odraslog. Analiticar moze da udje u ovo kruzno kretanje, ali on zaista ne moze ni u kom slucaju da zauzme mesto roditelja, jer dete ne oseca nikakvu potrebu da u njemu nadje substitutivnu figure. Stav koji analiticar zauzima prema detetu- druga prepreka. Analiticar dece, po A. Frojd, treba da bude sve drugo nego “ senka “, jer ne bi smeo da se odrekne svoje vaspitne uloge I dete mora da prepozna “ normativni “ cilj terapijskog poduhvata- u ovim uslovima Ani F. analiticar izgleda kao osrednji transferni objekt. Ona smatra da u normalnim uslovima transfer nije ni moguc, ni pozeljan. Smatra da se nikada ne radi o pravoj transfernoj neurozi, I to zbog spoljasnjih uslova, stvarnog prisustva roditelja odgovornih za vaspitanje deteta.

184. Koji su najcesci fantazmi u decijim strahovima? Najveci broj strahova odnosi se na osobe za koje je dete najvise vezano. Brige u vezi sa mogucom povredom osobe za koju je dete najvise vezano. Strah d ace osobe otici I nece se vratiti. Briga d ace ga neki dogadjaj, odlazak u bolnicu ili da se izgubi, odvojiti od osobe. Strah da ostane samo ili bez osobe za koju je najvise vezano.

185. Uzroci mucanja kod dece? Mucanje- poremecaj govora. Poremecaj ritma, odnosno vremenskog odnosa izmedju glasova u reci I recenici. Takav govor karakterise cesto ponavljanje ili produzavanje glasova, slogova ili reci ili naizmenicno ceste pause. Pocetak mucanja- obicno pre 10 godine ( izmedju 3 I 5 godine ). 1% dece nastavlja da muca I u adolescenciji- reakcije na dogadjaje koji su stresni, burni. Uzroci- psihogeni, organski I fizioloski. Pored nedovoljnog biolaskog sazrevanja, zona mozga odgovornih za govor, vaznu I procentualno znatno cescu ulogu u nastanku poremecaja imaju emocionalni faktori- strs u detinjstvu I uticaj okoline u kojoj dete zivi, ( poremeceni odnos dete- roditelji ). Mucanje- najcesce traumatogenog porekla. Jedan od uzroka je strah od sopstvene agresije, pa rec zaostaje u grlu. Cesci kod decaka. 186. 3 fantazma po Frojdu? Fantazam falusne majke. Fantazam kastracije. Fantazam roditeljskog koitusa ( Frojd ga je nazvao primitivna scena ).

187. Polimorfno kod Frojda? Uobicajeno je govoriti o polnim nastranostima u odraslom dobu. Kako je I dete normalno “ polimorfno “ nastrano od rodjenja ( Frojd ) I njegova polnost podvgnuta igri “ parcijalna pulzija “ povezanih sa drugim delovima tela, a ne genitalnim, izraz polna nastranost retko se koristi u decijoj psihijatriji sem izuzetno, kad su odstupanja u psihoseksualnom razvoju upadljivo velika I konacna.

188. Retroaktivno ponistavanje? Zeli da se ucinik ili misli nesto sto bi bilo odredjeno da izbrise, na realan ili imaginaran nacin, akciju ili misao koju Ja ne prihvata. 189. Vilijamsov sindrom?

Retka genetska bolest okarakterisana slaboscu za obuzdavanje, mentalne retardacije, teskoce ucenja. Karakteristican izraz lica ukljucuje natecenost u predelu ociju, kratak nos, siroka usta, pune usne I malu bradu. Osobe sa Vilijamsovim sindromom mogu takodje da imaju dug vrat, spustena ramena, nizak rast, ogranicenu pokretljivost zglobova I pokretljivost kicme. Vise od 50% dece sa vilijamsovim sindromom imaju deficit paznje, oko 50% specificne fobije, koje se odnose na strah I jake zvuke. Vilijamsov sindrom je nedovoljno poznato I kompleksno oboljenje. Lecenje se bazira na individualnim simptomima. Prognoza je varijabilna.

190. Razlike izmedju neuroza I poremecaja nedostatka? Rakamije razlikuje jasno “ patologiju nedostatka “ I neurozu. Lisavanje uvodi u zivot pocetni nedostatak iz koga osoba mora da izadje oslabljena. Ni u jednom momentu nije rec o konfliktnoj frutraciji zelja koju bi bilo moguce nadvladati I cije bi resenje moglo sacinjavati, kao u Edipovom kompleksu, iskustvo koje oblikuje. Frustracija koja se srece kod neuroticara nikad nema autenticnost ovog stvarnog nedostatka. Odnos sa roditeljima, koji je ocuvan u neurozi, dozvoljava osobi da udje u imaginarnu igru identifikacija, a iz koje je, afektivno liseno dete upravo ukljuceno.

191. Ciljevi terapije roditelja? Terapeut nece moci da deluje ako smenjuje autoritativnog, omrznutog ili zeljenog roditelja, ili ako preuzima ulogu “ zamene roditelja “. Njegova akcija postaje uspesna samo pod uslovom da pomogne roditeljima da postanu svesni psihickih konflikata koji napinju njihov odnos sa detetom I da ih rese. -

Da ne smenjuje. Da ne preuzima ulogu zamene.

Cilj- da se dopre do neuroticne strukture licnosti.

192. Kako Melani Klajn opisuje tezinu neuroze?

Merila -

Inhibicija epistemofilnih tendencija- nedostatak radoznalosti. Potiskivanje imaginarnog zivota ( maste, fantazma ). Nesposobnost da se podnese frustacija- osujecenje. Prekomerno prilagodjavanje na vaspitne zahteve. Nacin afektivnih odnosa deteta sa njegovom porodicnom okolinom I drugom decom.

193. Rakamije- cinioci afektivnog lisavanja? Posledice su najozbiljnije na psiholoskom planu, osecajni nedostatak, moze prouzrokovati zastoj razvoja. Osiromasenje Ja, koliko u narcistickim, toliko u objektnim investiranjima- nesposobnost da se voli, da bude voljen- Rakamije je oznacio nazivom apersonalizacija I sto objasnjava teskocu koju imaju takva deca da se identifikuju sa roditeljskom slikom. Javljanje I razvoj sindroma afektivnog nedostatka podleze izvesnim ciniocima. Postoje odojcad, koja su osetljivija nego druga deca na afektivni nedostatak. Rakamije to objasnjava- neka odojcad imaju narocito nizak prag tolerancije na frustraciju. Posledice su u toku 2 I 3 tromesecja I utoliko su jace ukoliko je odojce od pocetka bilo podizano u boljim uslovima. Posledice su utoliko jace ukoliko traju duze. Materinstvo, pod raznim vidovima, omogucava obnavljanje normalnog razvoja. Ali nakon izvesnog trajanja isuvise jake jacine, rano odvajanje stvara nepovratne posledice koje mogu biti ravne konacnom defektu buduci da je deficit utisnut u samu strukturu psihosomatske organizacije osobe.

194. Seksualne pulzije? Razlikuju se od drugih instiktivnih pulzija po sledecim osnovnim crtama Opire se prilagodjavanju osobe na spoljasnje uslove. Suprotno gladi ili zedji seksualne pulzije su pogodne za auto- erotsku gratifikaciju I mogu biti cak trenutno zadovoljene halucinarnom gratifikacijom. Izmicu neumoljivoj nuznosti potrebe za prilagodkjavanjem na sredinu. Objekt seksualne pulzije se pojavljuje u potpuno posebnom polozaju u odnosu na druge objekte. Cesto menjaju objekat zadovoljenja. Izvor se takodje menja.

195. Ovo pitanje je vec bilo

196. Isto

197. Razlike izmedju neuroza I poremecaja navika? Poremecaji navika spadaju u specificne poremecaje ponasanja koji se karakterisu ponavljanim postupcima koji nemaju jasnu racionalnu motivaciju. Pacijent saopstava da je patolosko ponasanje povezano sa impulsima koje on ne moze da kontrolise. U neurozama konflikti su nesvesni, a o njihovom postojanju zakljucujemo na osnovu ponasanja pacijenta. Poremecaji navika Kockanje, Hiromanija, Kleptomanija, Trihotilomanija.

198. Indikacije za psihoanalizu po M. Klajn? M. Klajn se ne obraca strukturnim datostima u pravom smislu. Ona pristupa ovom problem koristeci druge kriterije, treba otkriti kako se nereseni konflikti odrazavaju na osobu, proceniti patogenu nabijenost strahom I inhibirajuci svojstva koja decija neuroza vrsi na psihicko funkcionisanje. Njen dijagnosticki postupak usmeren je na otkrivanje straha iza izvesnih simptoma, ali isto tako I na otkrivanje inhibicija nekih ponasanja. Indikacije- poremecaj sna, poremecaj ishrane, fobicni simptomi. Ona u prvom redu razmatra znake decijeg straha, insistirajuci na poremecajima sna, teskocama ishrane I fobicnim simptomima. Poremecaj sna- uznemireno, cesto prekidano uspavljivanje, rana budjenja ili nesposobnost da se spava u toku dana. Ove teskoce su ekvivalenti nocnih strahova I direktno su vezani za fantazme koji navode na strah. Poremecaji u podrucju ishrane mogu biti pokazatelji neuroze. Originalnost M. Klajn- tezi da proceni teskoce deteta, ne polazeci od simptoma, nego od promene svakodnevnih ponasanja. Ona privlaci paznju na izvesne inhibicije koje se odnose na aktivnost igranja. Indikacije ce biti sire I pod “ idealnim okolnostima “ M. Klajn ne vidi nikakvu prepreku za profilakticnu analizu sve dece. S prakticnog gledista psihoanaliza ce biti namenjena slucajevima gde decja neuroza ne moze naci spontano resenje.

199. Ga nema nigde

200. Psihoterapija podrske kod roditelja? Tezi da Umanji frustraciju I osecanje krivice, Podigne granicu tolerancije prema psihoterapiji njihovog deteta, Rasvetljava znacenje koje ova terapija ima za dete, kao I razocarenja I teskoce koje ona povlaci. Ova terapija podrazumeva razgovore koji odrzavaju, u promenljivom ritnu, trajni kontakt s jednim ili oba roditelja. Psihoterapija podrske Rec je o tome da ona ne kani da direktno razresi neuroticne probleme roditelja, vec da olaksa napetosti prouzrokovane psihoterapijom deteta. Tehnika u okviru ove terapije Case- work- omogucava ne samo da se uskladi ova terapija sa roditeljskim reakcijama, vec cesto da se ide izvan toga da bi se odrasla osoba suocila sa svojim vlastitim problemima.

201. Po Frojdu- odnos psihickog traumatizma I neuroza? Rodjenje kao osnovni raskid veze sa majkom. Odbijanje od dojke kao prelaz od potpune decije zavisnosti do prvog stupnja nezavisnosti. Edip kao napustanje objekta prve zelje bitni su za licnost ili neurozu. Ali, ukoliko naknadne okolnosti ozive afekt vezan za ove traumatizme koji su stvorili neresena I potisnuta konfliktna neraspolozenja, oni ce u buduce moci da igraju patogenu ulogu. Majka koja preterano zasticuje. Afektivno odsutan otac. Prikriveni sado- mazohisticki odnosi roditeljskog para. Mogu najverovatnije biti patogeniji nego traumaticni dogadjaji koji se rado optuzuju da bi se objasnile neuroze.

202. Koji sup o Frojdu pokazatelji jacine Ega? Jak Ego dobro kontrolise unutrasnju realnost I uspesno izlazi na kraj sa slepim prohtevima licnosti, uz istovremeno jasno I pouzdano uvidjanje spoljnih mogucnosti. Mehanizmi odbrane jakog Ega takodje su postojani I uspesno funkcionisu.

203. Tip “ kao da “ na granici neuroze I psihoze? Vrsta shizoidnog karaktera, koji se ponasa kao da ima uobicajene emocionalne odgovore na situacije. Helen Dojc- tip na granici neuroze I psihoze U svakom novom objektnom odnosu zive kao da u stvari zive sopstveni zivot I izrazavaju sopstvena osecanja, misljkenja I shvatanja.

204. Na kom stadijumu dete prelazi iz aktivnog u pasivni psihoseksualni razvoj? Na sado- analnom stadijumu.

205. Identifikacija kao razvojna pojava I mehanizmi odbrane, poistovecivanje? Razlikuje se od ostalih mehanizama, ali se priblizava sublimaciji. Predstavlja normalni razvojni process kojim se licnost trajnio menja I obogacuje. Kao mehanizmi odbrane u uzem smislu te reci javlja se u razlicitim modalitetima, medju kojima su najpoznatiji Poistovecenje sa voljenim objektom, Poistovecenje sa izgubljenim objektom, Poistovecenje sa agresorom, Poistovecenje usled osecanja krivice.

206. Terapija autizma? Terapija igrom, Dinamicka psihoterapija,

Kognitivno- bihejvioralna terapija, Podrska roditeljima, Porodicna terapija. Bihejvioralna terapija Najefikasnija u ranoj primeni ( pre 4 godine ) sa decom ciji je IQ najmanje 50. Sistem nagrada- kazna. Autisticna deca slabo reaguju na sekundarne nagrade ili na socijalne nagrade. Bihejvioralna modifikacija pocinje psiholoskom edukacijom roditelja. Izabere se ciljno ponasanje ( obicno jednostavno, imitacija, pravljenje jednostavnih zvukova, ucenje jednostavnih znakova u komunikaciji ). Primarne nagrade imaju vecu moc za autisticnu decu. Tesko je izvesti bihejvioralnu terapiju jer ova deca tesko opserviraju I imitiraju I tesko komuniciraju.

207. Frojdovo ispitivanje Mali Hans? On neurozu odraslih dovodi u vezu sa decijom neurozom. Poziva se na edipov kompleks I kastracioni strah kada govori o jezgrovnom konfliktu koji strukturira neuroze I koji je moga biti ustanovljen u jednom klinickom slucaju, opservacija prve “ analize “ deteta, analize malog Hansa. Ova fobicna neuroza kod deteta od 5 godina, koju je Frojd analizirao oko 1906 godine omogucila mu je da proveri teorije o decijoj seksualnosti kako ju je on rekonstruisao pocevsi od materijala sakupljenog u toku analiza odraslih.

208. Terapija konverzije? Psihoterapija, Placebo, Hipnoza, Sugestija, Povecanje samopouzdanja, Sredstva za smirenje, Nagradjivanje, Relaksacija, Bih modifikacija, Porodicna terapija. 209. Kakvo je telesno Ja u snovima?

Pojam telesnog Ja je tesno vezan za Ja, pod kojim je Frojd podrazumevao onaj deo Ja koji potice od sopstvenih samooperacija o sebi, nasuprot percepcija o spoljasnjim objektima. Telesna sema je neurioloski termin za organske reprezentacije tela u mozgu, dok je slika sebi psiholoski termini za shvatanje koje self ima o vlastitom telu.

210. Enkompreza- tehnike lecenje? Medicinske Purgative ( lekovi za ciscenje ), klisteri I cepici u slucaju zatvora, Promena ishrane ( dijete ). Bihejvioralne intervencije Samokontrolisanje ( nadzor, pracenje ), Raspored ( plan ) koriscenja toaleta, Ohrabrivanje- nagradjivanje za cist ves, Ohrabrivanje- nagradjivanje za koriscenje toaleta, Time- out, Igra sa kupatilom, Biofidbek, Obelezavanje donjeg vesa. Porodicne intervencije- porodicna terapija, psihoterapija Terapija kroz igru, Kognitivno- bihejvioralna.

211. Bilo pitanje

212. Margaret Maler- priblizavanje kada se javlja I karakteristike, oralni stadijum?

Priblizavanje- zasnovano na iskustvu hranjenja. Uzajamne glad- sitost povecavaju uzajamnu prijemcivost deteta- majke.

kroz ponavljanje iskustva

Priblizavanje- pociva na simbiotskom odnosu izmedju deteta I majke. Kao period ovog odnosa javlja se strepnja, anakliticna depresija. Ovo se dovodi u vezu sa procesima individuacije I odvajanja koji se polagano odvijaju od 16 meseca do 3 godine zivota, kada se dozivljava I jaca

strepnja odvajanja. Moze se javiti simbioticna psihoza vec krajem prve godine, ali najcesce izmedju 3- 6 godine.

213. Setting u psihoterapiji? Terapeut unapred propisuje kakva ce terapija da bude, koliki ce biti broj seansi, koliko ce trajati, vrsi pripremu deteta I roditelja, odredjuje prostoriju koja ce uvek biti ista.

214. Averzivna terapija kod enkompreze, kada se koristi? Kazna time- out- ogranicavanje aktivnosti koje dete voli, uklanjanje lutke koju dete voli na neko vreme, teranje na odlazak u wc posle jela…

215. Koje su bolesti cesce kod devojcica, a koje kod decaka? Decaci Somnabulizam, Nocno mokrenje, Deficiranje, Samoubistvo uopste, Lupanje glavom, Tikovi, Hiperkinetika, Poremecaj ponasanja, Toksikomanija, Umna zaostalost, Neuroza ( 7- 11 ), Aspergerov sindrom. Devojcice Mentalna anoreksija, Gojaznost, Cupanje dlaka, Samoubistvo,

Neuroza ( 12- 19 godina ), Fobicna neuroza.

216. Kada je indikovana psihodinamska psihoterapija kod hiperkinetickog sindroma? Terapija igrom- psihodinamska psihoterapija Za mladju decu Resavaju nisko samopostovanje, Za anksiozne depresije, Ljutnju na autoritete, Frustraciju- agresiju. Za stariju decu Psihoterapija podrske. Ostali tretmani Najcesce su dijetalne intervencije. Neki smatraju da ovaj sindrom nastaje zbog trovanja olovom- zivom, ali redje da ovo trovanje moze izazvati trajnije simptome.

217. Intervencije porodice I skole kod konverzivnog poremecaja? Porodicne intervencije Porodica utice na izbor organa kroz koji ce da se izrazi symptom. Porodica smatra da deca normalno funkcionisu u socijalnom, intelektualnom I emocionalnom smislu I opiru se psihickim objasnjenjima konverzivnih simptoma, imajuvelika ocekivanja od dece, veliki nivo kontrole I vezanosti- ispod kojih su osnove osecaja straha da porodica nije bas “ normalna “, ted a bi na izgled sve izgledalo normalno I pod kontrolom, izbegavaju se jake emocije, a ispoljavaju se kroz telesne probleme.

218. Govor autisticne dece? Postoje opste kocenje I retardacija govora ili kognitivnog razvoja. U terapiji dete se nagradjuje za bilo koji vokalni izgovor, samo kad vokalizacija sledi govor terapeuta. Dete

uci fenomene I njihovo kombinovanje u reci, razumevanje kad dete nauci da simbolizuje upotrebu reci, recenice.

219. Koje poremecaje svrstavamo u poremecaje navika? Patolosko kockanje ( kod odraslih ), Podmetanje pozara, patolosko ( piromanija ), Patoloska kradja ( kleptomanija ), Patolosko cupanje kose ( trihotilomanija ).

220. Asperger- ov sindrom? Javlja se ostecenje reciprocnih socijalnih interakcija, koje karakterisu autizam, uz skucene, stereotipne, repetitivne repertoare, interesovanja I aktivnosti. Vecinom se javlja kod decaka. Razlika od autizma- ne postoji opste kocenje I retardacija govora ili kognitivnog razvoja. Najveci broj individua je normalnih intelektualnih sposobnosti, ali su cesto trapavi. Javlja se obicno u 3 godini. Visok IQ, Blaza forma autizma, Ostecena desna hemisfera, Okolina im smeta, Gledaju iznad osobe koja im prica, Govor- monolog, Nije uspostavljena simbioza sa majkom, Razlika od Kannerovog- motoricki razvoj pre verbalnog.

221. “ Holding “ tehnika kod autisticne dece I misljenje kod autisticne dece? “ Holding “ Integracija potpomognuta drzanjem I posedovanjem Dete stice osecanje stalnosti I povezanosti zavisno od sigurnostikoju mu obezbedjuju osobe- roditelji. Holding stiti dete od fizioloskih opasnosti. Zastitu pruzaju roditelji danju I nocu, svojom brigom za telo deteta ( preko dodira, temperature, sluha I vida ).

Poremecaj toka I sadrzaja misljenja ili se misljenje nece razviti, ako se razvije Sporost, Lepljivost, Prekidanje misaonog toka, Raspadanje misli I misljenja, Cudne misli, zaokupljenost…

222. Fetisizam kod dece I odraslih? Zamenjivanje majcinog tela, posebno grudi nekim predmetom koje dete prisvaja. Zamenu nalazi, kako u svom telu, tako u delovima tela drugih, ali I u delovima odece. Kod odraslih- fetis ima znacajnu, cak kljucnu ulogu I predstavlja cilj polnog mastanja. U razvoju fetisistickih sklonosti I u izboru fetisa znacajnu ulogu igra majka. Ona se nesvesno opire odvajanju od deteta I odbija da zadovoljava njegove telesne potrebe u pojedinim fazama razvoja. Ovaj odnos moze da bude uspostavljen kroz fetis koji predstavlja ili deo tela majke ili njeno telo u celini. Fetis- ima simbolicko, ali I stvarno znacenje I obezbedjuje detetu gledanje ili dodirivanje I mirisanje. Dete se na taj nacin oslobadja rusilackih, sadistickih poriva koje ulaze u ovaj fetisobjekat.

223. Razlika primarne I sekundarne identifikacije na kom uzrastu se javlja? Pocetno, prvobitno poistovecivanje ( primarna identifikacija ) deluje u prvim mesecima I godinama zivota ( do 3 godine ), najosetljivijoj I uticajima najpodloznijoj razvojnoj fazi. Dete pounutri I prvobitno se poistoveti sa culnim utiscima ( dodirnim, zvucnim, svetlosnim, mirisnim… ) koja mu upucuju najblize osobe. Sveukupna davanja roditelja su prvo pounutrena, a zatim, ili istovremeno, poistovecena sa sobom. Ne treba zaboraviti da je ovo faza razvoja u kojoj ne postoji jasna granica izmedju Ja I Ne- Ja, unutrasnjeg I spoljasnjeg. U prvoj fazi sekundarne ili struktuirajuce identifikacije, od 3- 5 godine zivota nastavlja se uoblicavanje Ja I nad- Ja. Dete je u to vreme manje zavisnije od drugih objekata nego u fazi

primarne identifikacije. Donekle je uspostavljen unutrasnji nadzor nad unutrasnjim I spoljnim podsticajima I dete delimicno ili sasvim usvaja osecanja, ponasanja, govor, polni identitet… Nakon puberteta Ja , snabdeveno vlastitim iskustvima, manje se podvrgava spoljnim slikama I uzorima, a vise se oslanja na svoja vlastita iskustva, sisteme vrednosti I delom izgradjen JaIdeal. Ako su ucvrscenja u prethodnim fazama razvoja jaka, omladinac teze dostize ravan nezavisnosti I poistovecivanja I ostaje izrazito zavisan od roditeljskih slika.

224. Istorijat nastanka psihoterapije detinjstva I mladosti? Nema ga odgovor

225. Ima ga vec to pitanje

226. I ovo pitanje ga ima

227. Neka glupost

228. Razlike M.Klajn I A.Frojd u terapiji igrom? M. Klajn Obratila se igrama I crtezima da bi uspostavila kontakt sa detetom I tako mu dozvolila da izrazi svoje fantazme. Igra, kao prirodna aktivnost deteta, ucinila joj se kao idealan nacin, utoliko pre sto se kroz igru, nastavljenu u prisustvu analiticara, izrazava fantazmatski zivot. S jedne strane, igra izrazava nesvesne seksualne I agresivne fantazme, ali u obliku igranja koje je istovremeno simbolicko I aktivno. S druge strane, aktivnost igranja u prisustvu analiticara daje igri novo znacenje, posto je analiticar tu da pokaze da “ igra ide mnogo dalje od igre “ I da se on mesa u detetov odnos sa libidinoznim objektima. Oko igre se uspostavlja odnos analiticar- dete, koji zigose transferna situacija.

Dete se igra u prisustvu analiticara I njegova igra tako gubi svoju “spontanost”. Ono se igra u situaciji pred nekim ko ga nadgleda I ko mu govori. Tako igra vise nije spontana. A.Frojd Ne zadovoljava se jedino verbalnim materijalom I obraca se crtezima koje dete pravi u toku seansi. Ona ne porice da bi se igra mogla interpretirati simbolickim izrazima, ali ona podvlaci da- ako asocijacije koje stvara odrasli svedoce o voljnom naporu da se svesno ne precuti nista iz svog govora- igra deteta ne odgovara takvom stavu. Ona narocito osporava mogucnost da se interpretira material igre u okviru transferne situacije, I ova teskoca za nju predstavlja najvecu prepreku u analizi deteta. 229. Nesto tamo pise 230. Koje 3 faze prolazi dete u razvoju identiteta? Za M. Mahler strepnja je izazvana prekidom simbiotskog odnosa izmedju deteta I majke I dozivljavanjem nestajanja, a zatim procesima odvajanja I individuacije koji se odvijaju u 3 faze Priblizavanje, Individuacija (16 meseci ), Separacija.

231. Kada se dete igra u igri ozivljava sukobe taj mehanizam odbrane je? Prisila ponavljanja.

232. Nasledne karakteristike licnosti? Nasledljivost- heritabilnost Veca nasledljivost je za intelektualne sposobnosti Govorne, Saznajne, Prostorne I Asocijativne. Obelezja licnosti Energetski kapaciteti, Osecajnost,

Sklonost ka strepnji, Impulsivnost, Agresivnost.

233. Obelezja decijih objasnjenja? Finalizam, Animizam, Artificijelizam. Period konkretnih operacija- 7- 11 godina. Period formalnih operacija- 11, 14, 15 godina.

234. Faze, stupnjevi emocionalnog razvoja? Zavisnost razvoja- svaka sledeca faza razvoja zavisi I pod uticajem je one prethodne. Kriticki period- ako se neka funkcija ne razvija u vreme odredjenog perioda, bice tesko ili nemoguce da je dete stekne kasnije. Istovremenost- uskladjenost cinilaca sazrevanja I razvoja. 235. Karakteristike hospitalizma? Zaostajanje u senzo- motornom razvoju, Relativna nezainteresovanost za okolinu, Podloznost infekcijama, Autoerotske radnje.

236. Faze odvajanja ( Bolbi I Ana F. )? Faza protesta ( protestuje, ljuti se, nada se, place ), Faza ocaja ( nesreca, smanjenje nade, akutan bol ), Faza poricanja ( B. ), Faza povlacenja ( A. F. ). Nije mu stalo ni do cega, ni do koga.

237. Patoloska dejstva ranog afektivnog lisavanja- Ana F.? Psihosomatska stanja ( poremecaj sna, apetita, varenja ),

Regresija u razvoju nagona ( autoerotske aktivnosti, samopovredjivanje, agresijapljuvanje, grizenje, sutiranje ), Regresija u razvoju Ja ( gubitak govora, losa socijalna adaptacija ), Poremecaj u distribuciji libida ( veze se za detetovo telo- hipohondrijske smetnje, u sliku o sebi- omnipotencije, ideje velicine, autizam I potpuni prekid sa realnoscu ).

238. Redosled nadoknadjivanja sposobnosti? Senzomotorne, Govor, Emocionalnost.

239. Pasivnost u aktivnost? Tokom 2 godine zivota se javlja licna zamenica Ja koja predstavlja idetitet u punom smislu. Primarna zavisnost prerasta u primarnu nezavisnost.

240. Faze igre- Frojd? Drugo dete je smetnja, Postaje zanimljivo, ali ne kao ljudsko bice, Interesuje se za istu stvar I saradjuju, ali zbog zajednickog cilja ( 2- 3 godine ), Ceni se drugo dete kao licnost.

241. Mehanizmi odbrane? Pounutrenje/ introjekcija ( simbolicko unosenje u sebe ), Projekcija/ - ( simbolicko odbacivanje ) / pospoljenje, Poistovecenje/ identifikacija, Nazadovanje/ regresija ( svedoci o slabosti u organizaciji Ja ), Pomeranje, Potiskivanje, Negacija- psihoze sa sumanutostima I halucinacijama, Izolacija, Ponistenje, Obrtanje u suprotno ( reaktivna formacija ), Racionalizacija I intelektualizacija, Sublimacija.

242. Funkcije samoubilacke radnje? Funkcija poziva u pomoc, Funkcija samounistenja I kaznjavanja sebe, Funkcija kaznjavanja sebe, Funkcija sloma, Funkcija ucenjivanja, Funkcija bega, Funkcija sjedinjenja sa drugim. Hajm “ ordalicka “ funkcija izazov sudbine, Boziji sud, da/ ne.

243. Pol Federn? Osecanje Ja- osecanje jedinstva u pogledu trajnosti, bliskosti I uzrocnosti u iskustvu jedinke. Ja- subjekt- Ja- nosilac svesnosti, Objekt- self. Telesno Ja- slozeno osecanje koje ukljucuje sva motorna I culna osecanja u vezi sa sopstvenom licnoscu I deo je mentalnog osecanja Ja.

View more...

Comments

Copyright ©2017 KUPDF Inc.
SUPPORT KUPDF